You are on page 1of 77

Module 1: Fundamental Concepts of Pharmacology

A. Drug Definitions and Classification

Pharmacology
▪ “Pharmakon” - Greek word means “an active principle”
▪ “Logos” - Greek word means “knowledge”
▪ Deals with the study of drugs and their effects on living organism.
▪ It includes everything about drugs
✓ Sources, Properties, Preparations, Dispensing, Administration in the body, Drug-body
interaction, Effects & Mechanisms of action of drugs, Rational use of drugs (safety,
efficacy, economic aspects of drugs)
▪ “Clinical pharmacology” - the study of drugs in humans

Purposes of Pharmacology
Pure Science
▪ To determine the response of living organisms to chemical stimuli in the presence and absence
of disease (pain in surgical procedure)
▪ It attempts to correlate chemical structure with biological activity and to classify chemically
related drugs into groups (barbiturates / sulfonamides)

Applied and Practical Science


▪ To test drugs quantitatively and standardize them so they are available to the doctor and
patient in uniform and dependable form
▪ To determine how drugs, produce their effects on the animal body
▪ To develop new drugs
▪ To establish the rational and safe dosage.

Branches of Pharmacology
Pharmacognosy
▪ study of characteristics of natural drugs and their sources

Pharmacodynamics
▪ study of what drugs do to the body

Pharmacokinetics
▪ what the body does to drugs

Pharmacotherapeutics
▪ study of how drugs are used to treat disease

Toxicology
▪ study of poisonous effects of drug

Pharmacogenetics
▪ scientific discipline studying how the effect of a drug action varies from a predicted
drug response because of genetic factors or hereditary influence.
Drugs (dried)
▪ Chemical substances that affect living organism and does not act as food.
▪ Derived from French word “Drogue” [meaning a dry herb].
▪ “Any substance or product that is used or intended to be used to modify or explore
physiological system or pathological states for the benefit of the recipient”. (WHO)
▪ Beneficial or not

Medicines
▪ Therapeutic drugs or drugs for the prevention of diseases.
▪ Beneficial

Sources of Drugs
▪ Many drugs originate as
Natural products
✓ Plants, Animals, Minerals, Bacteria or fungi (microorganism)

Animals
✓ Liver – treatment of pernicious anemia, Thyroid – treatment of hypothyroidism, Insulin
– treatment of DM, Cortisone and ACTH – treatment of arthritis, asthma and other
diseases

Inorganic Minerals
✓ elementary substances (Oxygen, Iodine & Iron), free acids (boric acid & hydrochloric
acid, metallic hydroxides (AlOH), and salts (Epson salts & Mg trisilicate)

Synthetic Drugs
✓ pure chemical substances produced in the laboratory by the organic chemist.
✓ Man made
✓ Produce in the laboratory

Plants
1. Catharanthus roseus (Periwinkle)
▪ Alkaloids used as chemotherapeutic agents
2. Aloe vera (Sinkle Bible)
▪ Glycosides used for cardiac disorders
3. Azadirachta indica (Neem)
▪ Tannis used for skin irritations and infections

Significant plant-derived pharmaceutical products


Source plant Drug Indication
Salix app aspirin analgesic
anti-pyretic
cardiovascular
Atropa belladonna astropine anti-cholinergic
Pupil dilation
Digitalis lanata digoxin cardiotonic
Mucuna deeringiana (L)-dopa anti-parkinsonism
Diocorea deltoidei diosgenin anti-fertility
Papaver somniferum morphine analgesic and antitussive
codeline
colchicum autumnale colchicine anti-tumour
anti-gout
cinchona ledgeriana quinine anti-malarial
catharanthus roseus vinblastine
Animal
Animal Part Drug Use
Cow Pancreas Insulin Antidiabetic hormaone
Fish Sperms Protamine suphate Antidote of heparin
Pig Intestine Heparin Anticoagulants
Ox Lungs Heparin Anticoagulants

Mineral Sources:
Metallic and Nonmetallic sources:
➢ Iron is used in treatment of iron deficiency anemia
➢ Mercurial salts are used in Syphilis (bacterial inf.).
➢ Zinc is used as zinc supplement. Zinc oxide paste is used in wounds and
in eczema.
➢ Iodine is antiseptic. Iodine supplements are also used.
➢ Gold salts are used in the treatment of rheumatoid arthritis.

Synthetic Drugs
Prepared by chemical synthesis in pharmaceutical laboratories
➢ Sulphonamides
➢ Salicylates
➢ Barbiturates
➢ Benzodiazepines

MICROORGANISM SOURCE
 Bacteria, Fungi, Moulds imp source of many lifesaving drugs.
 These obtained from MO and used to kill Microorganisms.

DRUG MICROORGANISM
Penicillin Penicilium notatum
Chloramphenicol Streptomyces venezuelace
Griseofluvin Penicillin grisofullivum
Streptomycin Streptomyces griseus
Neomycin Strptomyces fradiae

NOTES:
 30 bacteria from Mt. Mayon
 Pharmaceutical companies - Unilab

Drug Insert
✓ detailed description of a product required to be included in the package
Content:
▪ Brand name -
▪ Generic name- 1st manufacture
▪ Description
▪ Clinical Pharmacology/Pharmacotherapuetics
▪ Indications and usage
▪ Contraindications
▪ Warning
▪ Precautions
▪ Over dosage
▪ How supplied
Prescription
✓ An order for medication duly signed by a medical practitioner. Content:
Descriptive client information/ patient information
➢ Name, Address, Age or Birthday

Date prescription
➢ Date in which the prescription is written by the physician

Superscription/Recipe
➢ The Rx symbols

Inscription
➢ Name and dosage strength of the prescribed medication

Subscriptions
➢ Dispensing instructions for the pharmacist

Signa/ Transcription
➢ Directions for the client

Special instruction
➢ Refill

Prescriber’s information
➢ Name, DEA number specialization, signature, Tel. no, address and
license no.

US BASE

Four Basic Parts of the prescription


1. Superscription Rx
2. Inscription Medication prescribed
3. Subscription Instruction to pharmacist
4. Transcription Direction to patient
5. Signature physician
➢ Prescription Blank for a Single Medication
➢ Prescription Blank for a Multiple Medications
http://www.mapharm.com/prescr_parts.htm

1. Physician Information
2. Patient Information
3. Date prescription was written
4. Superscription
5. Inscription
6. Subscription
7. Signa
8. Special instruction

B. Drug Standards and Drug Information

Pharmacopeia
▪ Official publication, containing a list of medicinal drugs with their effects and directions for
their use.

International Pharmacopeia
▪ first published in 1951 by the World Health Organization (WHO)
▪ Provides a basis for standards in strength and composition of drugs for use throughout the
world.

Food Drug Administration


▪ Created under the Department of Health to license, monitor, and regulate the flow of food,
drugs, cosmetics, medical devices, and household hazardous waste in the Philippines.
▪ Main goal is to ensure the health and safety of food and drugs made available to the public.
▪ created under Republic Act No. 3720, series of 1963, as amended by Executive Order 175, series
of 1987, otherwise known as the “Food, Drugs and Devices, and Cosmetics Act”, and
subsequently Republic Act No. 9711 otherwise known as “The Food and Drug Administration
Act of 2009”

Philippine National Formulary


▪ Integral component of the Philippine Medicines Policy which aims to promote rational use of
essential medicines among prescribers, dispensers, patient and consumers.
▪ It is a list of essential medicines deemed important to address the primary health care needs
of Filipinos.
▪ As such it is the basis of all government procurement of medicines and reimbursement of
drugs of the Philippine Health Insurance Corporation
▪ search for the copy of the PNF 2018: https://pharmadiv.doh.gov.ph/uncategorised/17-
philippine-national-formulary

USP – NF
▪ Combination of two compendium the United States Pharmacopeia (USP) and the National
Formulary (NF). Monographs for drug substances, dosage forms, and compounded
preparations are featured in the USP. Monographs for dietary supplements and ingredients
appear in a separate section of the USP. Excipient monographs are in the NF.
▪ Search the website below: https://www.usp.org/about https://uspnf.com
Drug Approval Process

Philippine Schedule of Dangerous Drugs

• Codeine
• Opium
• Heroin
• Alprazolam
• Diazepam
• Cannabis – marijuana
• Schedule 2 – Dexamphetamine
• Schedule 4 – Pentobarbital (seizures) psychotropic alprazolam
• Schedule 5 – nalbuphine

Pregnancy Categories
Pregnancy Category
FDA PREGNANCY CATEGORIES
• A - No risk to fetus. Studies have not shown evidence of fetal harm
• B – No risk in animal studies, and well-controlled studies in pregnant women are
not available. It is assumed there is littler to no risk in pregnant woman
• C – Animal studies indicates a risk to the fetus. Controlled studies on pregnant
women are not available. Risk versus benefit of the drug must be determined
• D – A risk to the human fetus has been proved. Risk versus benefit of the drug
must be used in life-threatening conditions
• X – A risk to the human fetus has been proved. Risk outweighs the benefit, and
drug should be avoided during pregnancy

OTC – Over the counter


• Category C – Tamiflu, Zoloft
• Category D - Chemotherapeutic – Cytoxan (for cancer)
• Category X – avoided throughout pregnancy – Accutane (severe acne) Lipitor
(cholesterol)

Drug Names
1. Official name
➢ Drug listed in the official publication.
➢ PNF – Philippine national formulary

2. Chemical name
➢ Identifies chemical structure of the drug and exact description of atoms and
atomic groupings
➢ Mefenamic acid – anthranilic acid

3. Generic name
➢ Name that was established when drug was first manufacture; written in lowercase
letters; Official name of a drug; each drug has only one generic name; Original
manufacturer is only company that can use generic name for the first 17 years;
proprietary name or common name.

4. Drug Names
➢ Brand name; Name under which the drug is sold by a specific manufacturer;
Spelling always begins with a capital letter; Also known as the Trade Name; Name
is owned by the drug company and no other company may use that name; Each
brand name carries a registered trademark symbol ®; Proprietary name; assigned
name from the manufacturer.

TYPES OF DRUGS
5. Prescription drugs
➢ “Caution” – Law prohibits dispensing without prescription.
➢ Antibiotic
➢ Amoxicillin

6. Non-prescription drugs
➢ Over the counter drugs (OTC) drugs.
➢ Paracetamol, ibuprofen
7. Dangerous drugs
➢ Refers to either regulated or prohibited drugs
➢ Philippine drug scheduling
➢ Amphetamines – hypnotic, regulated, special prescription
➢ Alprazolam

8. Regulated drugs
➢ Drugs which produces a physiological action similar to amphetamines and
hypnotic drugs
➢ Men amphetamine

9. Prohibited drugs
➢ Contains opium and Indian hemp whether natural or synthetic with the
physiological effect of a synthetic drug
➢ Cocaine, heroin

10. Illegal drugs (recreational drugs)


➢ Drugs with no therapeutic effect

C. Pharmacokinetics
PHARMACEUTICAL PHASE

Pharmaceutics
➢ The study of the ways in which various forms influence pharmacokinetic and
pharmacodynamic activities.

Disintegration
➢ The breakdown of a tablet into smaller particles

Dissolution
➢ The dissolving of the smaller particles in the GI fluid before absorption

Rate of Dissolution
➢ Time it takes the drug to disintegrate and dissolve to become available for the body
to absorb

▪ Drugs in liquid form are more rapidly available for GI absorption that are solids
▪ Drugs disintegrated and absorbed faster in acidic fluids with a pH of 1 or 2 rather than
in alkaline fluids.
▪ Alkaline drugs would become ionized and have difficulty crossing cell membrane
barriers.
What happens to:
✓ enteric Coated Drugs – for entering intestine. aspirin, potency C, omeprazole
(acid)
✓ crushing of drugs – over dosage or under dosage
✓ sustained-release drugs – time for absorption, rate , nifedipine (longer effect)
Note:
▪ The more rapid the rate of dissolution, the more readily the compound crosses the
biological membrane to achieve absorption.
▪ Oral drugs in liquid are readily available for absorption because they are already in
solution.

PHARMACOKINETICS
Pharmakon
➢ Greek translation for drug
Kinetics
➢ refers to movement

Pharmacokinetics
 Movement of drug across the body membranes to reach its target organ to achieve
drug action
 Pharmacokinetics discusses how a drug is: Absorbed, Distributed, Metabolized,
Excreted
 Pharmacokinetics is also concerned with a drug’s:

Onset of action
▪ Refers to the time interval from when the drug is administered to when its
therapeutic effect actually begins.
▪ Onset varies depending on the route of administration.

Peak concentration level


▪ Reached when the absorption rate equals the elimination rate
▪ center

Duration of action
▪ The length of time the drug produces its therapeutic effect.
▪ Gaano katagal yung effect sa katawan
A. ABSORPTION
➢ Covers a drug’s progress from the time it’s administered, through its passage to
the tissues, until it becomes available for use by the body.
➢ Most oral drugs absorbed into the surface area of the small intestine through the
action of extensive mucosal villi.
➢ GI membrane is composed mostly of lipid and protein therefore lipid soluble drugs
pass rapidly through the GI membrane
➢ Water soluble drug need a carrier (enzyme or protein) to pass the membrane
➢ Large particles pass through the cell membrane if they are non-ionized.
➢ Weak acid drugs are less ionized in the stomach

First Pass Effect / Hepatic First Pass


 Following absorption of oral drugs from the GI tract, it passes from the intestinal lumen
to the liver via the portal vein.
 In the liver, some drugs are metabolized to an inactive form and are excreted, thus
reducing the amount of active drug available to exert a pharmacologic effect.
 ost oral drugs are affected by first pass metabolism
 Liver major organ of the body – small intestine – liver – hepatic portal - metabolized

Bioavailability
 The percentage of active substances in drug that is absorbed and available to the
target tissues following administration.
 Oral drugs < 100% bioavailable
 Intravenous drugs is 100% bioavailable
 Factors that affect bioavailability: dosage form, route of administration, GI mucosa and
mobility, food and other drugs, changes in liver metabolism

➢ On cellular levels, drugs are absorbed by several means:


Passive Transport
▪ requires NO cellular energy because drug moves from an area of higher
concentration to one of lower concentration.
a. Diffusion
b. Facilitated diffusion
c. Osmosis
Active Transport
▪ REQUIRES cellular energy to move the drug from an area of lower
concentration to one of higher concentration.
a. Endocytosis
b. Protein pumps
c. Exocytosis

Rate of Absorption of Drug


Slow Absorption
▪ Orally (swallowed), through mucous membrane, topical / transdermal, rectally

Faster Absorption
▪ Parenterally (injection), Inhaled through lungs, Directly into the brain
General Principle:
➢ Drugs that are lipid soluble and non-ionized are absorbed faster than the water soluble
and non- ionized. The faster the absorption, the quicker the onset, the higher the
addictiveness, but the shorter the duration

Factors influencing Absorption:


✓ Effects of pH
✓ Blood flow to the absorption site
✓ Total surface area available for absorption
✓ Contact time at the absorption site
✓ Expression of p-glycoprotein --- transmembrane transporter protein

Other factors:
✓ First Pass Effect
✓ Pain and Stress
✓ Stomach Content
✓ Dosage Forms
✓ Drug Combination
✓ Absorption: Solubility
▪ Water-soluble
▪ Lipid-soluble
✓ Ionized (have electrical charge)
✓ Crosses through pores in capillaries, but not cell membranes

2. DISTRIBUTION
➢ Process by which the drug is delivered to body tissues and fluids.
➢ Drug must pass though the capillary wall
➢ Blood brain barrier – very effective in keeping drugs from getting into the central nervous system or
CNS –limits movement of drug molecules into brain tissue

Blood Brain Barrier


▪ Semipermeable membrane in the CNS that protects the brain from foreign body.
▪ This is especially important when treating infections of the brain such as meningitis,
encephalitis, or brain abscess
▪ Medications must be able to penetrate the blood brain barrier
▪ Medications usually given intravenous

Factors that affect distribution of absorbed drug:


➢ Blood Flow
➢ Solubility
➢ Protein Binding
* Organs that receives large supply of blood: Heart, Liver, Kidneys
* Distribution to other internal organs, skin, fat and muscle is slower. In Drug Solubility:
➢ Lipid Soluble drugs easily cross through cell membranes
➢ Water Soluble drugs can’t cross through cell membranes

In Drug Binding:
Highly Protein-Bound
✓ Drugs that are 89% bound to protein
Moderately Protein-Bound
✓ -30% to 60% bound to protein
Low Protein-Bound
Note:
➢ Only free drugs (drugs not bound to protein) are active and can cause a pharmacologic
response
➢ Two highly protein bound drug given concurrently they compete for protein-binding sites
freer drug
➢ Liver and kidney disease (low serum protein)

3. METABOLISM or BIOTRANSFORMATION
➢ Process by which the body changes a drug from its dosage form to a more water-soluble form that
can be excreted.
➢ First Pass effect - can influence the effectiveness of orally administered drug, which pass through
the liver and partially metabolized before entering the systemic circulation.

➢ The liver’s primary mechanism for metabolizing drugs is via a specific group pf cytochrome P-450
enzyme. The level of these cytochrome P-450 enzymes controls the rate at which many drugs are
metabolizes.

Decrease
✓ Cytochrome P-450 the drug’s effect (including side effects) are increased
Increase
✓ Cytochrome P-450 the drug’s effects are decreased

Ways on How a drug is metabolized


➢ Most drugs are metabolized into inactive metabolites (product of metabolism), which are then
excreted.
➢ Other drugs are converted to active metabolites, which are capable of exerting their own
pharmacologic action. Metabolites may undergo further metabolism or may be excreted from
the body unchanged.

Prodrugs
✓ administered as inactive drugs which don’t become active until they’re
metabolized.
✓ E.G sulfasalazine – not active in its digested form, it should be broken down by
bacteria in the colon into 2 produces – 5ASA and sulfa pyridine before becoming
active as drug

Site of Metabolism:
✓ LIVER, Plasma, Kidneys, Membranes of Intestines

➢ Bioavailability of drug can be increased by prodrug. E.g. dopamine is the drug, levodopa is the
prodrug Levodopa can pass the BBB, gets converted to dopamine by dopa decarboxylase
➢ Prolong the duration. E.g. esters of phenothiazine have longer duration of action when
compared to phenothiazine
➢ Improve taste. E.g. clindamycin palmitate suspension
➢ Site specific delivery. E.g. Methenamine is a prodrug converted to formaldehyde by gastric pH
used as urinary antiseptic
Factors Affecting Drug Metabolism
• Route of administration
• Chemical Properties of drugs
• Dosage
• Age
• Genetics
• Diet
• Gender
• Diseases that reduce metabolism

Half-life (t ½)
➢ The time it takes for one half of the drug concentration to be eliminated.
➢ Metabolism and elimination affect the half-life of a drug

Table 1-2 HALF-LIFE OF 650 mg OF ASPIRIN

4. EXCRETION or ELIMINATION
➢ Elimination of drugs from the body.
➢ Routes of Drug Excretion: KIDNEYS, Lungs, Exocrine glands, Skin, Intestinal tract
➢ The kidneys filter free unbound drugs, water-soluble drugs, and drugs that are unchanged.
➢ The lungs eliminate volatile drug substances and products metabolized to carbon dioxide and water
➢ The urine pH influences drug excretion. Urine pH varies from 4.5 to 8.
➢ Acidic urine promotes elimination of weak base drugs, and alkaline urine promotes elimination of weak
acid drugs.

Laboratory Values
➢ Laboratory values reflecting function of liver and kidneys need to be looked at.
• BUN and Creatinine – kidney function
• LFT or liver function tests:
✓ ALT – alamine aminotransferase (elevated in hepatitis)
✓ AST or SGOT– aspartate aminotransferase – elevated in liver disease
✓ ALP – elevated in biliary tract disease
✓ Bilirubin levels – infants – gallstones in adults

Serum Drug Levels


➢ Laboratory measurement of the amount of drug in the blood at a particular time
➢ A minimum effective concentration (MEC) must be present before a drug exerts its action on
a cell.
➢ Serum drug levels indicate the onset, peak and duration of the drug action
➢ Need to check for serum level for drugs with narrow margin of safety (digoxin, aminoglycoside
antibiotics, lithium), to check to see if the drug is at its therapeutic level like seizure
medications, when drug overdose is suspected

Toxic Levels
➢ Excessive level of a drug in the body
• Single large dose
• Repeated small doses
• Slow metabolism which allows drug to accumulate in the body
• Slow excretion from the body by the kidneys or gastrointestinal tract
Serum half-life or elimination half-life
➢ Time it takes the serum concentration of the drug to reach 50%
➢ A drug with a short half-life requires more frequent administration
➢ A drug with a long half-life requires less frequent administration

Step 1: Absorption
▪ If administered topically it might move through the skin or a mucous membrane to
reach the bloodstream
▪ Drugs can pass through cells by either passive transport (no ATP required) or active
transport (ATP required)
▪ Bioavailability is the proportion of a drug that is successfully absorbed into systematic
circulation

Step 2: Distribution
▪ The way that drugs move through the bloodstream
▪ Tight binding prevents delivery
▪ Plasma protein
▪ A second drug with higher affinity for the protein can displace the first
▪ The blood-brain barrier prevents passage into brain tissue

▪ Psychotropic drugs (affecting the mind) can pass through


▪ Depends on anatomical barriers found in certain organs (blood-brain barrier, blood-
placental barrier, blood-testicular barrier)

Step 3: Metabolism
▪ Reactions that inactivate the drug and target for excretion
▪ Drugs taken orally are absorbed through the intestinal wall
▪ Drugs taken orally will enter the hepatic portal system
▪ First-pass effect: first pass of a drug through the liver
▪ Drugs are metabolized after reaching their target as well
Step 4: Excretion
▪ The drug or its remnants will finally exit the body
▪ Glands that produce saliva and sweat promote some excretion

D. PHARMACODYNAMICS
PHARMACODYNAMICS
➢ It is the study of drug mechanisms that produce biochemical or physiologic changes in the
body.
➢ It is the study of drug action on living tissue.
➢ It is concerned with the response of tissue to specific chemical agents at various sites on the
body.
➢ Drug response can cause a primary or secondary physiologic effect or both.
✓ The primary effect is desirable, and the secondary effect may be desirable or
undesirable

EFFECTS OF DRUGS
DOSE
➢ is the amount of medication given for the desired therapeutic effect?

Kinds:
Loading dose
➢ initial high dose given to patient to achieve minimum effect or elevate level of
drug in the blood

Minimum dose
➢ Minimum amt. of drug that will cause a therapeutic effect

Therapeutic dose
➢ Amount of drug given that will cause a therapeutic effect.

Maximum dose
➢ Largest amount of drug that will produce a desired effect without producing
symptoms of toxicity

Toxic dose
➢ Amount that will produce harmful side effects or symptoms of poisoning

Lethal dose
➢ Dose that causes death

Maintenance dose
➢ Dose required to keep the drug blood level at a steady state in order to
maintain the desired effect
General Properties of Drugs
1. Drugs do not create new responses but alter existing physiologic activity.
2. Drugs in general exert multiple actions rather than a single effect.
3. Drugs result from a physicochemical interaction between the drug and a functionally
important molecule in the body.
4. Drugs interact to a receptor called agonist.

Terms Indicating Drug Action


1. Indication
➢ a list of medical conditions or disease for which the drug is meant for

2. Actions
➢ a description of cellular changes that occur as a result of drugs

3. Contraindications
➢ a list of all conditions for which the drug should not be given

4. Cautions
➢ a list of conditions or types that warrant closer observation for specific side effects
when given the drug

5. Side effects/adverse reaction


➢ list of possible unpleasant or dangerous secondary effects other than the desired
effects (ototoxicity, nephrotoxicity, photosensitivity, hepatotoxicity)

6. Interactions
➢ a list of drugs or foods that may alter the effect of the drug usually should be given
during same course of therapy

Dose Response and Maximal Efficacy


➢ Dose response is the relationship between the minimal versus the maximal amount of drug
dose needed to produce the desired drug response.
➢ Some patients respond to a lower drug dose, whereas others need a high drug dose to elicit
the desired response.
➢ The drug dose is usually adjusted to achieve the desired drug response.
➢ All drugs have a maximum drug effect (maximal efficacy).
➢ E.g. maximum efficacy of morphine > tramadol

Onset, Peak, and Duration of Action


a. Onset of action is the time it takes to reach the minimum effective concentration (MEC) after
a drug is administered.
b. Peak action occurs when the drug reaches its highest blood or plasma concentration.
c. Duration of action is the length of time the drug has a pharmacologic effect.
Mechanism of Drug Action
➢ It is the means by which drug produce an alteration in function in their action known as the
mechanism of action.

Receptor
o is an active cellular site, which a drug can interact to produce a pharmacological
response.
o Drug receptor interaction
o Produced their effect by one of the following:
1. Drug receptor interaction

2. Drug enzyme interaction

▪ Drug enzyme interaction is similar to drug receptor interactions. The


drugs resemble the natural substances, bind enzymes and cause
changes in their activity.

3. Non-specific interaction
Receptor Theory
➢ Drugs act through receptors by binding to the receptor to produce (initiate) a
response or to block (prevent) a response.
➢ The activity of many drugs is determined by the ability of the drug to bind to a specific
receptor.
➢ The better the drug fits at the receptor site, the more biologically active the drug is.

Agonists and Antagonists


Agonists
➢ Drugs that produce a response E.g. epinephrine stimulates (the response) beta1 and
beta2
Antagonists
➢ Drugs that block a response E.g. atropine blocks histamine2

Nonspecific and Nonselective Drug Effects


Nonspecific
➢ Drugs that affects various sites E.g. drugs that affect cholinergic receptors
(bethanechol)
Nonselective
➢ rugs that acts on different receptors E.g. chlorpromazine (NE, Dopamine, Ach,
histamine receptors)

Classification of Drug Action


a. Agonist
➢ Those with complete attachment and response
➢ Drugs that interact with the receptor and will stimulate a response.

Affinity
• the propensity of a drug to bind or attach
Drug Efficacy
• drug ability to initiate activity as a result of binding; refers to a drug maximal
effectiveness (measured by patient’s vital sign, body weight)
Therapeutic drug level
• Refer to drug level that provide adequate action but minimal adverse
• E.g. heroin, oxycodone, methadone, hydrocodone, morphine, opium

b. Antagonist
➢ Drugs that attach to a receptor but not stimulate a response.

c. Partial Agonist
➢ Drug that interacts with a receptor to stimulate response but inhibit other responses.

d. Pure antagonist
➢ Drugs with no pharmacological action of their own but interact with a receptor to
inhibit or prevent the action of agonist.
➢ Agonists facilitate receptor response
Full Agonist
1. Codeine
2. Fentanyl
3. Heroin
4. Hydrocodone
5. Hydromorphone
6. Levorphanol
7. Meperidine
8. Methadone
9. Morphine
10. Oxycodone
11. oxymorphone

Partial Agonist
1. Buprenorphine
2. Butorphanol
3. Pentazocine
4. Tramadol

Mixed Agonist
1. Buprenorphine
2. Butorphanol
3. Nalbuphine
4. Pentazocine

Antagonist
1. Naloxone
2. Naltrexone
Important Drug-Drug Interactions

Categories of Drug Action


1. Stimulation
➢ rate of cell activity or secretion increases;
Depression
➢ cell activity and function of a specific organ are reduced

2. Replacement E.g. insulin

3. Inhibition or killing of organisms E.g. penicillin

4. Irritation E.g. laxatives

Therapeutic Index and Therapeutic Range (Therapeutic Window)

Therapeutic index (TI)



➢ Estimates the margin of safety of a drug through the use of a ratio that measures the effective
(therapeutic) dose (ED) in 50% of people (ED50) and the lethal dose (LD) in 50% of people
(LD50)
➢ The closer the ratio is to 1, the greater the danger of toxicity.
➢ TI = LD 50
ED50
➢ Drugs with a low therapeutic index have a narrow margin of safety
➢ Drug dosage might need adjustment, and plasma (serum) drug levels need to be monitored
because of the small safety range between the ED and LD.
➢ Drugs with a high therapeutic index have a wide margin of safety and less danger of
producing toxic effects
➢ Plasma (serum) drug levels do not need to be monitored routinely for drugs with a high TI.
Therapeutic range (therapeutic window) of a drug concentration in plasma
➢ Level of drug between the minimum effective concentration in the plasma for obtaining desired drug
action and the minimum toxic concentration (the toxic effect).
➢ When the therapeutic range is given, it includes both protein-bound and unbound portions of the drug.
➢ If the therapeutic range is narrow, such as for digoxin (0.5 to 1 ng/mL), the plasma drug level should
be monitored periodically to avoid drug toxicity.
➢ Monitoring the therapeutic range is not necessary if the drug is not considered highly toxic.

Peak and Trough Drug Levels

Peak drug levels


➢ Indicate the rate of absorption of the drug
➢ If the peak is too low, no therapeutic effect is achieved.
➢ Peak drug level is the highest plasma concentration of drug at a specific time.
➢ Peak drug levels indicate the rate of absorption.
➢ If the drug is given orally, the peak time might be 1 to 3 hours after drug administration.
➢ If the drug is given IV, the peak time might occur in 10 minutes.
➢ If a peak drug level is ordered, a blood sample should be drawn at the proposed peak time,
according to the route of administration.
Trough drug levels
➢ Indicate the rate of elimination of the drug.
➢ The trough drug level is the lowest plasma concentration of a drug, and it measures the rate
at which the drug is eliminated.
➢ Trough levels are drawn immediately before the next dose of drug is given, regardless of route
of administration.

Note:
✓ Peak and trough levels are requested for drugs that have a narrow therapeutic index
and are considered toxic
✓ If either the peak or trough level is too high, toxicity can occur.

Loading Dose
➢ Initial large dose of the drug that is given when immediate drug response is desired
➢ It is given to achieve a rapid minimum effective concentration in the plasma.
➢ After a large initial dose, a prescribed dosage per day is ordered.
➢ E.g. Digoxin (Digitek, Lanoxicaps, Lanoxin), a digitalis preparation, requires a loading dose
when first prescribed. Digitalization is the process by which the minimum effective
concentration level for digoxin is achieved in the plasma within a short time.

Drug Interactions
➢ Occurs when the action of one drug is altered by the action of another drug.

Pharmacokinetic and pharmacodynamics


➢ With pharmacokinetic drug interactions, one drug affects the absorption, distribution,
metabolism, or excretion of another.
➢ With pharmacodynamic drug interactions, two drugs have interactive effects in the
brain.
➢ Either type of drug interaction can result in adverse effects in some individuals.
➢ In terms of efficacy, there can be several types of interactions between medications:
cumulative, additive, synergistic, and antagonistic.

Types of Interactions
1. Cumulative
➢ The condition in which repeated administration of a drug may produce effects
that are more pronounced than those produced by the first dose
➢ Repeated admn. Of slow excreted drug
➢ Digoxin, emetine, heavy metals
➢ Cumulative effects desired: phenytoin in epilepsy
➢ Passive cumulation: remain deposited in bones without toxic effects- LEAD;
Toxic once in blood
➢ Liver and kidney impairment: non-cumulative drugs also cumulate

2. Additive
➢ The effect of two chemicals is equal to the sum of the effect of the two
chemicals taken separately.
➢ 1+1=2
➢ Occurs when two or more “like” drugs are combined. Example (Codeine with
acetaminophen work together to produce better pain control
➢ May be intentional or may unintentionally cause harm. Example (Alcohol and
anxiety medications can both cause sedation, taken together they greatly
increase the risk of sedation even to the possibility of death.
3. Synergistic
➢ The effect of two chemicals taken together is greater than the sum of their
separate effect at the same doses
➢ 1+1=3
➢ A synergistic effect occurs when two or more “unlike” drugs are used
together to produce a combine effect
➢ Consider the following example:
✓ A beneficial synergistic effect occurs when two different types of
antibiotics that work in very different ways are combined, such as
penicillin G and an aminoglycoside antibiotic

4. Potentiation
➢ The drug that has no effect enhances the effect of the second drug
➢ 0+1=2
➢ Combined action of two drugs
➢ “Enhanced” synergism
➢ Common examples:
✓ Benzos and Alcohol
✓ Benzos and Ketamine
✓ Opiates and Ketamine
✓ Clonidine and Timolol
✓ Warfarin and Amiodarone
✓ SMZ/TMP and Warfarin

5. Antagonistic
➢ The effect of two chemicals taken together is less than the sum of their
separate effect at the same doses
➢ Drug inhibiting the effect of another
➢ 1+1=0

6. Drug incompatibility
➢ Occurs when two drugs mixed together in a syringe produce a chemical
reaction, so they cannot be given together.
➢ 1+1=X
➢ Types of incompatibility: precipitation, color changes, gas evolution

Drug and Food Interaction


➢ May increase or decrease absorption
➢ May produce dangerous adverse effects

Common Response to Medications


1. Desired Effect
➢ Therapeutic response is achieved

2. Side Effects
➢ Physiologic effects not related to desired drug effects.
➢ All drugs have desirable or undesirable side effects.
➢ Even with a correct drug dosage, side effects occur and are predicted.
➢ Side effects result mostly from drugs that lack specificity
➢ The occurrence of these expected but undesirable side effects is not a reason
to discontinue therapy.
➢ Role of the nurse: patient teaching to report any side effects (dosage
adjustments, changing to a different drug in the same class of drugs, or
implementing other interventions.
Note:
✓ The occurrence of side effects is one of the primary reason’s patients stop
taking the prescribed medication

3. Adverse effects/reactions
➢ More severe than side effects.
➢ Range of untoward effects (unintended and occurring at normal doses) of
drugs that cause mild to severe side effects, including anaphylaxis
(cardiovascular collapse).
➢ Always undesirable.
➢ Need to be reported and documented because they represent variances from
planned therapy

4. Idiosyncratic response
➢ Strange unique or unpredicted reactions

5. Allergic/Hypersensitivity reaction
➢ An antigen-antibody reaction the release of histamine causing provoking
symptoms like itching or swelling of the skin.

6. Anaphylactic Response
➢ Severe formed of allergic reaction that is life threatening.

7. Toxic Effect/Toxicity
➢ Predictable adverse drug effects when plasma levels of a drug exceed the
upper limit of the therapeutic range.
➢ It can be identified by monitoring the plasma (serum) therapeutic range of the
drug.
➢ When the drug level exceeds the therapeutic range, toxic effects related to
overdosing or drug accumulation

8. Tolerance
➢ Exists when there is a decrease physiologic response to the repeated
administration of a drug or chemical related substance.
➢ In contrast, tachyphylaxis refers to a rapid decrease in response to the drug. In
essence, tachyphylaxis is an “acute tolerance.” Drug categories that can cause
tachyphylaxis include narcotics, barbiturates, laxatives, and psychotropic
agents

9. Tachyphylaxis
➢ Rapid decrease in response to the drug.
➢ Acute tolerance
➢ E.g. narcotics, barbiturates, laxatives, and psychotropic agents.

Note:
✓ Role of the nurse: need to distinguish tolerance of the patient to the drug

10. Resistance
➢ Occurs when the drug fails to exert its intended effect.

11. Dependence
➢ Occurs when the client needs the drug to function
12. Placebo effect
➢ Psychological benefit from a compound that may not have the chemical
structure of a drug effect.
➢ Effective in approximately one third of persons who take a placebo compound

Safety and Efficacy


o Safety is how many and what kinds of adverse effects are associated with the
drug.
o An adverse reaction may require immediate action.
o Be alert to complaints from patient after starting new drug.

When a patient complains a drug is not working, it may be due to any of the following:
➢ Patient does not understand how the drug works
➢ Dosage needs to be adjusted
➢ Therapeutic level has not been reached
➢ Wrong drug was prescribed
➢ Some drugs work better with one patient than another
➢ Some forms of drugs work better
➢ The trade name drug may work even if the generic does not

Nursing Considerations
1. Under NO circumstances should such a mixture be administered to the patient until the safety
of the mixture can be assured.
2. Before combining two drugs and solutions every attempt must be made ascertaining the
stability and safety of the mixture with the pharmacist and see package insert for more
information.

PHARMACOTHERAPEUTICS
➢ Refers to the use of drugs to treat disease or produce a desired effect

Therapeutic Methods:
1. Drug therapy
➢ treatment with drugs.

2. Diet therapy
➢ treatment by diet; such as low salt for patients with hypertension.

3. Physiotherapy
➢ treatment with natural physical forces such as water, light and heat.

4. Psychological therapy
➢ identification of stressors and methods to eliminate or reduce stress/ or the use of
drugs.
5. Empiric therapy
➢ based on practical experience rather than on pure scientific data.

6. Acute therapy
➢ for patient that is critically ill and requires acute intensive therapy.

7. Maintenance therapy
➢ for patients with chronic conditions that don’t resolve.
8. Supplemental or Replacement therapy
➢ to replenish or substitute for missing substances in the body.

9. Supportive therapy
➢ which doesn’t treat the cause of disease but maintains other threatened body systems
until the patient’s condition is resolved.

10. Palliative therapy


➢ used for end-stage or terminal diseases to make the patient as comfortable as possible.

Factors affecting Pharmacotherapeutics


➢ Patient’s disease
➢ Route of administration
➢ Patient’s body size, weight, sex, past medical condition
➢ Psychological and emotional stress
➢ Drug tolerance and dependence

Pharmacotherapeutics – Drug Categories


1. Their action on the body
2. General therapeutic effect
3. Body system affected

Pharmacotherapeutics – Indications and Labeling


➢ indications – used to identify purpose or reason for using a drug.
➢ FDA – approved indications which must be part of labeling
➢ Multiple uses of a drug are possible.

1. Symptomatic treatment
➢ used to relieve disease symptoms.

2. Preventive drug
➢ help the body avoid disease.

3. Diagnostic drugs
➢ help the physician whether the disease is present.

4. Curative drugs
➢ eliminate the disease.

5. Health maintenance drugs


➢ help keep the body functioning normally.

6. Contraceptive drugs
➢ prevent pregnancy.
Kinds of Therapy
1. Acute
➢ improve life threatening or serious condition

2. Empiric
➢ give until other tests prove another therapy is appropriate

3. Maintenance
➢ maintain a condition

4. Palliative
➢ reduce the severity of a condition or pain

E. Factors Influencing Responses to Drugs

F. Drug Legislation of Controlled Substances, Generic Drugs, Orphan drugs and Over-the –Counter Drugs

a. Republic Act 9165


➢ Comprehensive Dangerous Drug Act of 2002

b. Republic Act 6675

c. Republic Act 10747


➢ Rare Diseases Act of the Philippines
➢ Orphan drug refers to any drug or medicine used to treat or alleviate the symptoms of
persons afflicted with a rare disease and declared as such by the DOH upon
recommendation of the National Institutes of Health (NIH). (Article 2, Section 4)
➢ Orphan product refers to any healthcare or nutritional product, other than a drug or
medicine, including, but not limited to, diagnostic kits, medical devices and biological
products, used to prevent, diagnose, or treat rare diseases and declared as such by the
DOH upon recommendation of the NIH. (Article 2, Section 4)
➢ SEC. 16. Permit for Restricted Use of an Orphan Drug/Orphan Product. — Any person
may import any orphan drug/orphan product for compassionate use: Provided, That
they secure a compassionate special permit from the Food and Drug Administration
(FDA)… (Article 5, Section 16)
➢ read for list of rare disease and with corresponding orphan drug / products approved

d. Republic Act 9502


➢ Universally Accessible Cheaper and Quality Medicines Act of 2008
➢ read for the list of over the counter drugs in the Philippines

Nursing Considerations Related to Over-the-Counter Drugs


➢ Always read the instructions on the label.
➢ Do not take OTC medicines in higher dosages or for a longer time than the label states.
➢ If you do not get well, stop treating yourself and talk with a health care professional.
➢ Side effects from OTCs are relatively uncommon, but it is your job to know what side effects might
result from the medicines you are taking.
➢ Every person is different, therefore responses to medicines may also be different from one another.
➢ OTC medicines often interact with other medicines, and with food or alcohol, or they might have an
effect on other health problems you may have.
➢ If you do not understand the label, check with the pharmacist.
➢ Do not take medicine if the package does not have a label on it.
➢ Throw away medicines that have expired (are older than the date on the package).
➢ Do not use medicine that belongs to a friend.
➢ Buy products that treat only the symptoms you have.
➢ If cost is an issue, generic OTC products may be cheaper than brand name items.
➢ Avoid buying these products online, outside of well-known Internet insurance company sites, because
many OTC preparations sold through the Internet are counterfeit products.

Parents should know the following special information about using OTCs for children:
➢ Parents should never guess about the amount of medicine to give a child. Half an adult dose may be
too much or not enough to be effective. This is very true of medicines such as acetaminophen (Tylenol)
or ibuprofen (Advil), in which repeated overdoses may lead to poisoning of the child, liver destruction,
or coma
➢ If the label says to take 2 teaspoons and the dosing cup is marked with ounces only, get another
measuring device. Don't try to guess about how much should be given.
➢ Always follow the age limits listed.
➢ Always use the child-resistant cap, and relock the cap after use.
➢ Throw away old, discolored, or expired medicine or medicine that has lost its label instructions.
➢ Do not give medicine containing alcohol to children.
AOl 1

1. Identify the preferred focus of nursing care in maternal and child health
➢ Family-centered approach

2. In formulating an effective nursing care plan for patient Hinata who is on her 3rd trimester of pregnancy. Nurse
Sakura is considering all factors that might affect Hinata such as her living conditions, neighbors, workplace,
accessibility to health care facilities. Based on this statement, determine the philosophy of maternal and child
health nursing that nurse Sakura is considering
➢ Community centered

3. Nurse Mae is collecting all the required health data of her patient like immunization status, present and past
history of illness, subjective and objective cues. Nurse Mae is performing which of care for her pediatric
patient?
➢ Assessment

4. Nurse Jokha was assigned to do systemic investigation of the problem they are experiencing in their ward
concerning their nursing practice. Based on this, Nurse Jokha is expected to perform:
➢ Nursing research

5. Which of the following standard of professional performance is being applied to nurse Luna when the quality
and effectiveness of her nursing practice is being evaluated?
➢ Performance appraisal

6. Nurse Christian provides health education for his patient together with the family who just delivered a healthy
baby boy. The focus of his teaching is about dealing on having a new member in the family. Identify the nursing
theory that Nurse Christian is applying in his nursing care
➢ Adaptation model

7. Distinguish the standard of professional performances Nurse Luna is applying when she is contributing to the
professional development of her co-staff nurses.
➢ Collegiality

8. Which of the following statement is true regarding the role of maternal and child health nurse?
➢ Considering the family as a whole and as a partner in care when planning or implementing or
evaluating the effectiveness of care
➢ Serves as an advocate to protect the rights of all family members, including the fetus
➢ Serves as an important resource for families during childbearing and childrearing.

9. Nurse Tals was assigned to manage a maternity clinic. The role given to nurse Tals can be identified as:
➢ Managerial function

10. You received a patient who is second month pregnant, it is expected for you to provide nursing care under
which term of pregnancy?
➢ 1st trimester

11. After providing the care of nurse Totep to his patient, he makes sure that he continuously monitors the
outcome and use improvement methods based on the need. The QSEN competency can be identified as:
➢ Quality improvement

12. From being a novice, nurse Christian can now be considered an expert maternal and child health nurse after
gaining more knowledge and experiences in his practice. Nurse Christian is a good example of which theory
of nursing?
➢ Caring, clinical wisdom and ethics in nursing practice
13. Part of the framework in maternal and child health nursing is to apply certain competencies to improve the
quality of nursing care and build knowledge, skills and attitude. You are correct if you identify this framework
as:
➢ Quality and safety education for nurses

14. After identifying the nursing diagnosis, nurse make needs to develop strategies of care that will prescribe
interventions to obtain her expected outcome. This pediatric standard of care is known as:
➢ Planning

15. As per Kaila’s record, she is being given care under the stage of puerperium. Distinguish among the following
the possible condition of Karla
➢ Karla is on 3 weeks post-delivery

16. Nurse Dwight was invited in the community to provide health education to pregnant mothers like importance
of antenatal checkup, proper diet and personal hygiene. Determine the role nurse Dwight is performing as
maternal and child health nurse.
➢ Educational function

17. Nurse Aimee is caring for woman during a complication for pregnancy and her interventions are to return
patient to wellness. This phase of health care nurse Aimee is applying is known as:
➢ Health restoration

18. Distinguish among the following phases of health care nurse Aimee is applying when she is educating her
patient regarding the importance of immunization
➢ Health maintenance

19. Which of the following statements is true about SDG 3 good health and wellbeing?
➢ By 2030, reduce the global maternal ratio less than 70 per 100,000 live births
➢ By 2030, end preventable deaths of newborns and children under 5 years of age with all countries
aiming to reduce neonatal morality to at least as low as 12 per 1000 live births and under 5 mortality
to at least as low as 25 per 1000 live births
➢ Significant strides have been made in increasing the life expectancy and reducing some of the
common killers associated with child and maternal mortality.

20. Nurse Luna is making sure that she is evaluating her own nursing practice in relation to professional practices
standards and relevant status and regulation. Nurse Luna is performing which standard of professional?
➢ Performance appraisal

21. Which of the following is the major focus of nursing practice in maternal and child health?
➢ Childbearing and childrearing families

22. Which among the following is true about evidence-based practice?


➢ It integrates best current evidence for delivery of optimal health care.
➢ It is conscientious, explicit, and judicious use of current best evidence to make decision about the
care of patients
➢ The evidence can be a combination of research clinical expertise, and patient preferences or values

23. Nurse Luna is considering taking up master of science in nursing. Her plan can be considered under which
standard of professional performance?
➢ Performances approval

24. Nurse Totep is fully aware that his patient is the source of control and his fully partner n providing care
➢ Patient centered care
25. Differentiate obstetrics from pediatrics
➢ Obstetric means child while pediatrics means to keep watch
➢ None of the above

QUIZ 1:
1. Why must the nurse always ask a patient about his or her use of any herbal supplements or botanicals?
➢ These substances can interact with a prescribed drug.

2. Which statement about agonist and antagonist drugs is true?


➢ Both agonist and antagonist drugs must interact with receptors to produce their intended
responses.

3. A patient has a deficiency of an enzyme that prepares a specific drug for elimination. When planning care for
this patient, the nurse remains alert for which response or issue?
➢ Toxic blood levels of the drug are more likely to occur.

4. A patient asks the nurse about drug interactions with OTC preparations. Which of the following statements is
a correct response of the nurse?
➢ “Read the labels carefully, and check with your health care provider.”

5. The nurse is explaining drug action to a student nurse. Which statement made by the nurse is true?
➢ “Drug particles not bound to protein are free drugs that can cause therapeutic effect.”

6. “Drug particles not bound to protein are free drugs that can cause therapeutic effect.”
➢ Superscription

7. Dr. Raymond prescribed Ranitidine 150 mg tab, 1 tab twice a day before food for 5 days, #10 tabs. Which of the
following information can be identified as the inscription part of the prescription?
➢ Ranitidine 150 mg tab

8. Which among the following information is the subscription part of the prescription?
➢ #10 tabs

9. Instruction to patient must be clearly stated in the prescription. Determine the signa part on the prescription
among the following information.
➢ 1 tab twice a day before food for 5 days

10. Some doctors are including the information about refill in their prescription. This information can be identified
as:
➢ Special Instruction

11. Which of the following is the list of essential medicines that address the primary health care needs of Filipinos?
➢ Philippine National Formulary

12. How long will it take for drug B to be tested for effectiveness among 250 real patients?
➢ 2 years

13. Drug C has been out in the market already after being given an approval by the FDA. But drug company is still
testing it for long-term effects and additional uses. Drug C can be identified under which phase of drug-
approval process?
➢ Phase 4
14. When a drug like Alprazolam has been identified with accepted medical use in the Philippines, having a low
potential for abuse with limited physical or psychological dependence; this drug can be seen under the list of:
➢ Schedule 4

15. Mrs. Lustre who is 4 months pregnant has been diagnosed with breast cancer. Her doctor is studying the
possibility of giving chemotherapeutic drugs (cancer treatment). These drugs have been proven with risk to
fetus but the risk versus the benefit of the drug must be determine especially in life-threatening condition.
Based on FDA pregnancy category, these drugs can be identified under:
➢ Category D

16. Mang Hery was not given an antibacterial drug, Augmentin 625 mg tab because he doesn’t have a prescription
drug. This particular drug is considered as:
➢ Prescription drug

17. While on duty, the doctor asked you to bring his special prescription to be used for diazepam (anxiolytic). You
are ware that this drug produces a physiological action similar to amphetamines and hypnotic drugs, therefore
it is considered as:
➢ Regulated drug

18. Which of the following drugs can have a faster effect due to its 100% bioavailability?
➢ Diphenhydramine 50 mg IV

19. All of the following are forms of active transport in drug absorption, EXCEPT:
➢ Diffusion

20. All of the following are TRUE about drug protein binding, EXCEPT:
➢ Kidney disease won’t affect serum protein level, therefore, no effect on drug protein binding.

21. The liver of the infant is said to be not fully develop, thus it can affect drug metabolism. This factor can be
identified as:
➢ Age

22. Levodopa is a prodrug that is being given to dopamine. Determine the effect of this product.
➢ Increasing bioavailability of drug

23. Patient Santy was given high initial dose of 900 mg Phenytoin to achieve the desired effect. The dose given
to Santy can be considered as
➢ Loading

24. All of the following are TRUE about general properties of drug, EXCEPT:
➢ Drugs create new responses that alter existing physiologic activity.

25. Determine the action of the drug epinephrine that’s stimulates the Beta 1 cells to aid in patient suffering
from difficulty of breathing by dilating the bronchioles.
➢ Agonist

26. Gentamycin (aminoglycoside antibiotic) belongs to which category of drug action?


➢ Inhibition or killing of organisms

27. Mang Isko always fails to complete his antibacterial/ antibiotic treatment regimen. Because of this, many
antibacterial medicines are not working anymore with him. This drug response can be documented as:
➢ Residence

28. Aling Minda was advised to refrain from eating foods high in fats due to her elevated cholesterol level. What
kind of therapeutic therapy is being applied to Aling Minda?
➢ Diet Therapy

29. Determine the kind of therapy applicable for Michelle that uses vibration for his back problem.
➢ Physiotherapy

30. Which of the following laws addresses the needs of Carlos who is suffering from maple syrup disease, which
is considered as rare disease?
➢ Ra 10747

QUIZ 1:
1. What major consideration allows a drug to be available over the counter rather than by prescription?
➢ The drug is safe when the directions for dosage and scheduling are followed.

2. In the Philippines, which agency is responsible for enforcing established standards for drug approval?
➢ Food and Drug Administration

3. Which statement about drug side effects is true?


➢ All drugs have side effects.

4. Which drug administration route is most likely to result in reduced drug bioavailability?
➢ Oral

5. Nurse Cathy administers 500 mg drug orally at 8:00 in the morning with a half-file of 3 hours. What is the
expected strength of drug at 11:00 AM?
➢ 250 mg

6. All of the following body tissues or fluids are routes of drug elimination, EXCEPT:
➢ Bone marrow

7. The nurse is describing to a patient the synergistic effects of two of his medications. Which statement by the
nurse is true about synergistic drug effects?
➢ A greater effect is achieved when two drugs are combined.”

8. The nurse is giving a large initial dose of a ceftriaxone (antibiotic) to rapidly achieve minimum effective
concentration in the plasma. Which of the following is the type of dosage that the nurse has done?
➢ Loading dose

9. Dr. Reyes prescribed Amoxicillin 500 mg cap, 1 cap every 8 hours for 5 days after food, #15 capsules. Which of
the following information can be identified as the inscription part of the prescription?
➢ Amoxicillin 500 mg cap

10. Which among the following information is the subscription part of the prescription?
➢ # 15 capsules

11. Instruction to patient must be clearly stated in the prescription. Determine the signa part on the prescription
among the following information.
➢ 1 cap every 8 hours for 5 days after food
12. Determine the basis for standards in strength and composition of drugs being use worldwide that was
published by World Health Organization (WHO).
➢ International Pharmacopeia

13. As per the information provided by the drug company, drug A is being tested already for safety among 70
healthy individuals. Based from this, drug A is already at the phase of:
➢ Phase 1

14. Cocaine has been categorized with no accepted medical use in the Philippines. Based on the Philippine
schedule of dangerous drug, this can be included under:
➢ Schedule 1

15. Folic acid, proven with no risk to fetus, has been prescribed to Mrs. Sanchez who is 5 months pregnant. This
drug can be classified under which pregnancy category by FDA?
➢ Category A

16. Cocaine and heroin as examples of drugs that contain opium and Indian hemp with a physiological effect of s
synthetic drug. They can be classified as:
➢ Prohibited drug

17. A drug molecule can be transported by engulfing it with its membrane using cellular energy. This can be
identified as:
➢ Endocytosis

18. The drug given to Patient Carla has accumulated in her body in excess that causes harmful side effects and
symptoms of poisoning to her. This drug dose can be identified as:
➢ Toxic

19. Due to the over usage of the drug, Manny always feels uneasy and cannot function properly without taking
that drug. This drug response can be considered as:
➢ Dependence

20. Which of the following laws regulates the use of dangerous drugs in the Philippines?
➢ RA 9165
Module 2: Nursing Process in Pharmacology

QSEN Competencies
➢ Quality and Safety Education for Nurses
➢ Developed by the American Association of Colleges of Nursing
➢ 6 Competencies needed to prepare nurses with the knowledge, skills and attitude needed to improve
workplace quality and safety within the health care system
1. Patient Centered Care (protecting rights)
2. Teamwork and Collaboration
3. Evidence based Practice
4. Quality improvement
5. Safety
6. Informatics

A. Assessment
➢ Systematic validation and documentation of information which is facilitated in every phase of the
nursing process.

2 Types of Data
1. Subjective Data
➢ Current health history (include problem with swallowing)
➢ Patient symptoms
➢ Current medications (OTC, herbal remedies, nutritional supplement)
✓ Dosage, frequency, route
✓ Patient knowledge about drug
✓ Patient adherence with regimen and reasons for deviations
✓ Patient expectation and perception of the drug effectiveness
✓ Patient knowledge of effects, drug reaction that need to be reported
✓ Drug allergies or reactions both past and present, reactions to food
and dye
✓ Use of tobacco products, alcohol, caffeine, and recreational drugs
(frequency and duration of use)

➢ Past health history


✓ Past illnesses, major injuries, drug therapy
✓ Mental status
✓ Medications saved from previous use, reason for saving them and
expiration date

➢ Patient’s environment
✓ Home safety
✓ Patient’s language and communication needs
✓ Patient’s ability and readiness to read and follow instructions
✓ Patient knowledge of specific drug storage requirements
✓ Availability, willingness and psychomotor ability to administer or assist
in the administration of medications, capabilities for activities of daily
living
✓ Household members
✓ Learning style preferences
✓ Readiness to learn
✓ Dietary patterns, cultural and economic influences, safety
✓ Financial resources or limitations
2. Objective Data
➢ Physical health assessment limitations in gross and fine motor control, hand and
joint range of motion, hand and finger muscle strength, visual impairment, and
ability to read labels and correctly measure dosages.
➢ Laboratory test results and diagnostic studies baseline data (needed for future
comparison)

➢ System organ assessment focusing on those affected by drug therapy


➢ identify high risk patients

✓ Essential component of health teaching: adherence to medication regimen


Reason for non-adherence
▪ Forgetfulness
▪ knowledge deficit
▪ side effects
▪ low self-esteem
▪ depression
▪ lack of trust in health care system,
▪ family problems
▪ language barriers
▪ high cost of medications
▪ Anxiety
▪ value systems
▪ lack of motivation.

B. Nursing Diagnosis
➢ Formulated based on the analysis of the assessment data

Common Nursing Diagnosis


1. Pain (acute and chronic)
2. Acute confusion
3. Ineffective health maintenance
4. Deficient knowledge
5. Noncompliance
6. Risk for injury
7. Ineffective self-health management
8. Readiness for enhanced knowledge

C. Planning
➢ Characterized by setting goals or expected outcomes, development of nursing interventions
➢ Qualities of effective goal setting: patient centered, clearly stated, acceptable to both patient and
nurse, shared with other health care providers, identifies component for evaluation

D. Intervention
➢ Activities includes: patient teaching, medication administration, patient care

1. Medication Administration
➢ Quality and safety drug administration
I. Forms and routes for drug administration
A. Oral Drugs (tablets, capsules, liquids)
✓ Not given to patient who are vomiting, lack of gag reflex, comatose
✓ Do not mix with large amount of fluid, contraindicated food
✓ Enteric coated and timed-release capsules must be swallowed whole
✓ Be aware of those drugs that should never be cut in half or crushed
✓ Administer irritating drugs with food
✓ Administer drugs on an empty stomach if food interferes with absorption
✓ Drugs given sublingually / buccally remain in place until fully absorb
✓ Use of child resistant cap
✓ Suspensions and emulsions need to be shaken before administration, elixir is not
shaken

✓ The bottom of the meniscus should be used to measure the desired dose of
medication.
✓ Once reconstituted, many liquids require refrigeration as storage

B. Transdermal / Topical
✓ Wash hands and wear gloves when administering transdermal patch and topical
dosage forms

C. Instillations
✓ Close eyes for 2 to 3 minutes after instillation of eye ointment
✓ Pull down and back on the auricle for a child younger <3 years old
✓ Pull up and back for on the auricle for adult
✓ When using spray to reach sinuses, proper head position is with patient looking
down at feet and spray tip aimed toward eye
✓ Keep head tilted backward for 5 minutes after instillation of nasal drops

D. Inhalations
✓ MDIs (metered dose inhalers) = hand held devices used to deliver medications for
asthma and bronchitis to the lower respiratory tract via inhalation
E. Nasogastric and Gastrostomy Tubes
✓ Check for proper tube placement before administering
✓ Pour drug into the syringe without plunger or bulb, release clamp, and allow
medication to flow in properly
✓ Flush tubing with 50ml of water or as prescribed
✓ Clamp tube and remove syringe
F. Suppositories (rectal / vaginal)
✓ Refrigerate before use (soften at room temperature)
✓ For rectal position: left side lying then breath through the mouth to relax the anal
sphincter
✓ Have patient lie on the side for 20 minutes after insertion
✓ Instruct the patient to retain the medication in place
✓ For vaginal position: lithotomy

G. Parenteral
Intradermal
➢ Do not massage after injection

Subcutaneous
➢ Do not aspirate
➢ Apply gentle pressure on the site to prevent bleeding or oozing into the tissue
and subsequent bruising and tissue damage (patient on anticoagulant
therapy)

Intramuscular
➢ Z-tract injection technique prevents leaking back into the SQ tissue; gluteal
site preferred

Intravenous
➢ If anesthetic agent is to be applied like EMLA, apply it 1 hour before the
procedure

Intraosseous
➢ Infusion into the bone marrow, IV access is not possible
➢ Removed within 24hours
II. Rights to medication administration
➢ Foundation of medication safety
➢ Right patient
✓ Use of identification bracelet

➢ Right drug
✓ Prescribed by the right person: physician, dentist, podiatrist, advanced
practice RN, physician assistant
✓ Component of drug order: date and time the order is written, drug name, drug
dosage, route of administration, frequency and duration of administration,
special instructors for withholding or adjusting dosage, signature of prescribe
or name if TO or VO (need to be signed within 24hours)
✓ Once the drug has been administered, the nurse becomes liable for the
predicted effects of that drug.
✓ To avoid drug error: drug label is read 3x (a.) time of contact with the drug, (b.)
before measuring the drug, (c.) after measuring the drug
✓ First dose, one-time, PRN medication orders should be checked against the
original order.

➢ Right dose
✓ It is based on the patient’s physical status, including renal and hepatic function,
and weight.
✓ Calculate the dose accurately

➢ Right time
✓ Administer at the appropriate time to maintain the therapeutic level
✓ Long half-life (OD), short half-life (several times per day)
✓ Before meals or after meals related to the absorption of drug
✓ Check if the patient is scheduled for diagnostic procedure e.g. endoscopy
✓ Check expiration date
✓ Antibiotics at even intervals throughout a 24hours period to maintain
therapeutic level

➢ Right route
✓ Necessary for adequate or appropriate absorption
✓ Assess the client’s ability to swallow before administering oral medications.
Make sure the patient is not NPO
✓ Do not crush or mix medications in other substances without consultation, mix
with sweet substance /formula milk
✓ Explain to patient if it needs to be mix with other substance
✓ Use aseptic technique during administration
✓ Stay with the patient until oral drug has been swallowed
➢ Right assessment
✓ Gather appropriate data before administration e.g. vital signs, serum blood
level of drug

➢ Right documentation
✓ Need to record the following: name of drug, dose, route, time and date,
nurse’s initial or signature
✓ Document patient’s response to the following drugs: opioids, non-opioid
analgesics, sedatives, antiemetic, unexpected reaction to the drug (GI
irritation, skin sensitivity)
✓ Document after drug is administered.

➢ Right education
✓ To include therapeutic purpose, expected result of the drug, possible side
effects of the drug, a dietary restrictions or requirements, skill of
administration and laboratory test result monitoring
✓ Principle of informed consent
✓ Informed patient and family – critical to preventing medication error

➢ Right evaluation
✓ Appraisal of drug’s therapeutic and adverse effects
✓ Important aspect of patient safety

➢ Patient’s right to refuse


✓ Patient can and do refuse to take medication
✓ Determine reason for refusal and take reasonable measure to facilitate patient
taking the medication.

III. Nurse’s Right when administering Medications 6 Rights for Nurses who administers
medication
1. Right to a complete and clear order
2. Right to have the correct drug, route, dose dispensed
3. Right to have access to information
4. Right to have policies to guide in safe medication administration
5. Right to administer medication safely and to identify problem in the system
6. Right to stop, think and be vigilant when administering medications

IV. Culture of Safety


➢ Possible causes of medication errors: increase in the number of drugs available;
violation of the rights of nursing medication administration; lack of drug knowledge;
memory lapses; transcription, dispensing, or delivery problems; inadequate
monitoring; distractions; overworked staff; lack of standardization; equipment
failures; inadequate patient history; poor interdepartmental communication
➢ Safety risks for medication administration
✓ Splitting pill
✓ Buying drugs from the internet
✓ Crushing / Not to crush: Do not crush any medication that has the suffix ER, SR,
XR, SA, CR, MR, XL
✓ For oral dosage forms that should not be crushed, search on
https://www.ismp.org/recommendations/do-not-crush
✓ Look alike and sound alike drug names
➢ Pregnancy Category
➢ Search https://www.ismp.org/recommendations/error-prone-abbreviations-list for the
list of error-prone abbreviations
➢ For JCI standards abbreviations, search on
https://www.jci.org/kiosks/publish/abbreviations
➢ Search https://www.jointcommission.org/resources/news-and-multimedia/fact-
sheets/facts- about-do-not-use-list/ for JCI DO NOT USE LIST
➢ Behaviors to avoid during medication administration

V. Medication Reconciliation
➢ Medication reconciliation process of comparing a list of all medications that a patient
is currently taking with a list of new medications that are about to be provided
➢ It is conducted whenever a patient undergoes a transition in care in which new
medications may be ordered, or existing orders may be changed.
➢ Transition in care: hospital admission, discharge, transfer to another facility
➢ Five steps in medication reconciliation
1. Develop a list of current medications
2. Develop a list of medications prescribed
3. Compare the medications on the two lists
4. Make clinical decisions based on the comparison
5. Communicate the new list to appropriate caregivers and to the patient

VI. High Alert Drugs


➢ High-alert drugs cause significant harm to the patient when given in error
➢ Major effect on the patient’s organs: cardiac, respiratory, vascular, and neurologic
systems, sympathetic and parasympathetic nervous systems
➢ Specific high-alert medications as listed by the Institute for Safe Medication Practices
(ISMP) include: epinephrine, subcutaneous; epoprostenol, IV; insulin; magnesium
sulfate injection; methotrexate, nononcologic oral use; opium tincture; oxytocin, IV;
nitroprusside sodium for injection; potassium chloride concentrate for injection;
potassium phosphate injection; promethazine, IV; and vasopressin, IV or IO, all forms
of insulin, subcutaneous and IV, are considered high-alert medications.
➢ For additional classes and categories of high-alert medications, visit the Institute for
Safe Medication Practice’s website at http://www.ismp.org/.
➢ These lists are provided to reduce the risk of errors, but specific strategies can optimize
safety when dealing with high-alert drugs:
1. Simplify the storage, preparation, and administration of high-alert drugs.
2. Write policies concerning safe administration.
3. Improve information and education.
4. Limit access to high-alert medications.
5. Use labels and automated alerts.
6. Use redundancies (automated or independent double-checks).
7. Closely monitor the patient’s response to the medication (possibly the
most important step
2. Patient Teaching
➢ Patient education – ongoing, complex, multifaceted process and a dynamic interaction
between nurse and patient in which both communicate information and emotions.
➢ Focus: health promotion and specific skills to optimize patient health status
➢ Need to include family members (psychological support, actually administer drug, observe
effectiveness and side effects of drug therapy, implement other changes)

➢ Suggested topics to pharmacotherapeutics:


General:
✓ importance of adherence, pregnant client (always consider physician’s
advice), laboratory test that needs to be monitored

Self-administration:
✓ consider psychomotor skills, prescribed route of administration, clear
instructions, teaching more than one person for “backup”

Diet:
✓ food to include and avoid

Side effects:
✓ instruct to report unusual symptoms immediately, instructions on how to
minimize side effects, advise related to expected changes in urine and stool,
dizziness related to orthostatic hypotension

Cultural considerations:
✓ consider awareness of culture expectations

➢ Multiple medications
➢ Preparation of 1day’s or 1 week’s supply of medication
➢ Use of medication dispensers to sort medications
➢ Use of recording sheet, alarm reminder device

3. Patient Care
I. Developmental needs of pediatric patients
Stranger anxiety (infant):
✓ act to instill a sense of safety and security in the infant

Hospitalization, illness or injury viewed as punishment (3 to 6 years):


✓ allow control where appropriate, obtain child’s view of situation, encourage
positive relationship, expression of feelings in acceptable manner and
activities, include family or support persons

Fear of mutilation (3 to 6 years):


✓ explain procedure carefully, use less intrusive routes, allow child to play
injection to a doll or stuffed animal

II. Geriatric patients


➢ Polypharmacy
➢ Adherence and non-adherence to medication
E. Evaluation
➢ Determination of how well goals are met
➢ Nursing interventions are revised and teachings are focus to improve goals

F. Recording and Reporting


➢ Documentation is an essential component of all phases of the nursing process.

A. Guidelines Documenting Telephone and Verbal Orders


✓ Ask the prescriber to speak slowly and clearly
✓ If not familiar with the medication, ask the prescriber to spell it out
✓ If the drug, dosage is inappropriate to the client, ask the physician question
✓ Write the order down or enter into a computer on the physician’s order form
✓ Read the order back to the prescriber. Use words instead of the abbreviations.
✓ Make sure the prescriber verbally acknowledges the read back.
✓ Record date and time and indicate it was a telephone order (TO) sign name and credentials
✓ When writing a dosage always put a number before a decimal (e.g. 0.5mL) but never after
a decimal (e.g. 6mg)
✓ Write units (e.g. 5units)
✓ Transcribe the order
✓ Follow agency protocol about the prescriber’s protocol for signing telephone orders (e.g.
within 24hours)

B. Drug Administration of Opioid


✓ Sign out the drug on the appropriate form.
✓ Verify the amount of drug in the container before giving it.
✓ Have another nurse document the activity and observe the activity in case there is a waste
or need discard part of an opioid dose.

At the end of your shift:


✓ Record the amount of each opioid on the opioid control sheet while the nurse beginning
her shift counts the opioids out loud.
✓ Sign the opioid control sheet only if the count is correct. Have the other nurse countersign.
✓ Identify and correct any discrepancies before any nurse leaves the unit.
✓ If the discrepancy can’t be resolved, follow the employer’s policy for reporting this and file
an incident report. An investigation will follow.

C. Drug Administration – STAT Order


✓ Include the name of the person who gave the order, why the order was given, and the
patient’s response to the drug. In the MAR, write the drug’s name, dosage, route, and time
given.

D. Drug administration – Adverse Effect of…


✓ Nurses play a key role in reporting adverse drug effects events.
✓ Reporting adverse effects helps ensure the safety of drugs regulated by the Food and Drug
Administration (FDA).
✓ The FDA’s Medical Products Reporting Program supplies health care professionals with
MedWatch forms on which they can report adverse events.
✓ Complete a MedWatch form when you suspect that a drug is responsible for: death, life-
threatening illness, initial or prolonged hospitalization, disability, congenital anomaly, need
for any medical or surgical intervention to prevent a permanent impairment or an injury.
✓ Also, promptly inform the FDA of product quality problems, such as: defective devices,
inaccurate or unreadable product labels, packaging or product mix-ups, intrinsic or extrinsic
contamination or stability problems, particulates in injectable drugs, product damage.
✓ Report only the adverse event or the problem with the drug.

E. Patient Refusal to take the medication


✓ Notify his doctor and describe the event in his chart.
✓ By documenting the refusal, misinterpretation that the drug is omitted by mistake will be
avoided.
✓ Record the date and time of entry.
✓ Document that the patient refused to take his prescribed drugs and the reason, assuming
he tells the reason.
✓ Record the name of the refused drugs and the time they were due. Include any explanations
given on the indications for the drugs and why they were ordered for the patient.

F. Drug Administration: Withholding Ordered Drug


✓ Under certain circumstances, a prescribed drug can’t or shouldn’t be given as scheduled.
✓ E.g. withhold a stool softener for a patient with diarrhea, a patient who is scheduled for a
test that requires him to not take a certain drug, antihypertensive drug however the patient
is with low blood pressure
✓ If a drug is withheld, notify the doctor.
✓ Document the date and time the drug was withheld, the reason for withholding the drug,
the name of the doctor notified, and the doctor’s response.
✓ If the doctor changed a drug order, record and document the new order a nd the time it
was carried out.
✓ Document any actions taken to safeguard the patient.

G. Medication Error
✓ Medication errors are the most common, and potentially the most dangerous, errors.
✓ Mistakes in dosage, patient identification, or drug selection by nurses have led to vision loss,
brain damage, cardiac arrest, and death.
✓ A medication event report or incident report should be completed when a medication error
is discovered.
✓ The nurse who discovers the medication error is responsible for completing the medication
event report or incident report and for communicating the error to the patient’s doctor and
the nursing supervisor.
✓ In the nurse’s note, describe the situation objectively and include the name of the doctor
noti fied, the time of notification, and the doctor’s response.
✓ Avoid the use of such terms as “by mistake,” “somehow,” “unintentionally,”
“miscalculated,” and “confusing,” which can be interpreted as admissions of wrongdoing.
✓ Document the medication error on an incident rep ort or medication event report

H. Medication Reconciliation
✓ Reconciling medications is a process that develops an accurate, up-to-date medication list
for patients at admission and then compares that list against the doctor’s admission orders.
✓ Any discrepancies in the patient’s medications will be brought to the attention of the doctor
and changes made to the orders as necessary.
✓ The process is designed to promote communication and information transfer during patient
transfer and prevent errors, such as omissions, duplications, dosing errors, or drug
interactions.
✓ The ordering physician, the registered nurse, and the pharmacist all share accountability for
accurate medication reconciliation.
✓ Reconciliation of medications should occur within 24 hours of patient admission and at
every transition of care, such as a change in setting, service, practitioner, or level of care.
✓ Complete facility’s standardized form for reconciliation of medications.
✓ This form should include the dosage, dosing frequency, date and time of last dose, and the
purpose of each medication.
✓ It should also include the source of this information (patient, family member, caregiver, or
medication bottle) and any patient allergies.
✓ Reconciling medications is assisted by completion of the facility’s standardized forms for
admission, transfer, and discharge of the patient.
✓ For accurate medication reconciliation after completion of the admission form, obtain
information from the patient, the patient’s family, and the facility pharmacy.

AOL 2:
1. Noncompliance related to forgetfulness A. Assessment

2. Current health history A. Assessment

3. Goal setting C. Planning

4. Patient’s environment A. Assessment

5. Action to accomplish goals D. Implementation

6. Drug allergies and reactions A. Assessment

7. Referral

8. Patient/significant other education

9. Use of teaching drug cards D. Implementation

10. Laboratory test results A. Assessment

11. Productive cough A. Subjective Data

12. Pain in left ear A. Subjective Data

13. Lab values B. Objective Data

14. Nausea A. Subjective Data

15. Heart rate B. Objective Data

16. Right route C. Medication given IM as prescribed

17. Right patient F. Verification of patient ID

18. Right time J. Drug given at the time prescribed

19. Right documentation G. Nurse charts that patient pain was decreased after drug administration

20. Right assessment A. Measurement of a patient’s apical pulse


21. Right drug H. Patient receives the prescribed medication

22. Right dose B. Amount of medication given as prescribed

23. Right to education D. Teaching a patient about possible side effects of the medication

24. Right to refuse E. The patient refuses to take medication

25. Right evaluation I. Nurse checks blood pressure following blood pressure medication
administration

26. Drugs poured by others A. Do not administer

27. Patient states that drug is different than usual A. Do not administer

28. Bad-tasting drugs first, then pleasant-tasting drug

29. Drugs transferred from one container to another A. Do not administer

30. Drugs with date and time opened and your initials on label B. Do administer

QUIZ 2:
1. Cigarette smoking
➢ Subjective

2. Fasting blood glucose level is 115 mg/dl


➢ Objective

3. Taking Vit C daily


➢ Subjective

4. Doing regular exercises


➢ Subjective

5. Blood pressure of 120/70 mmHg


➢ Objective

6. Giving aspirin after meal to prevent gastric irritation.


➢ Right time

7. Insulin was injected subcutaneous for proper absorption


➢ Right route

8. Check for patient’s identification bracelet


➢ Right patient

9. Verifying the medicine in the medication card and MAR


➢ Right drug

10. The patient will be free from hyperactivity is included in which phase of the nursing process?
➢ Planning
11. The patient has been diagnosed with angina and hypertension and has been started on medication. Instruct
patient to avoid caffeine-containing beverages is included in which phase of the nursing process?
➢ Intervention

12. The patient has been prescribed a diuretic to treat hypertension and is currently taking an over-the-counter
allergy medication. Sleep pattern disturbance is included in which phase of the nursing process?
➢ Nursing Diagnosis

13. The nurse prepares and administers prescribed medications during which phase of the nursing process?
➢ Intervention

14. What is the best way for the nurse to make sure that the right patient is receiving a prescribed drug when
the patient is alert and oriented?
➢ Have the patient state his or her name and birth date

15. When is it acceptable for the nurse to take a verbal order from the prescriber before giving a drug to a
patient?
➢ In an emergency situation such as a cardiac arrest

16. What is the most important role of the nurse in preventing drug errors?
➢ Always following the ten rights of drug administration

17. A sublingual drug is administered by placing the drug in what part of the body?
➢ Under the tongue

18. What administration technique does the nurse use to give a 2-year-old child ear drops?
➢ Pull the earlobe down and back

19. What must the nurse be sure to tell the patient after a vaginal drug is administered?
➢ Remain lying down for 10 to 15 minutes after taking this drug

20. When giving a drug to a patient who is awake but confused, what is the best way for the nurse to identify
the patient?
➢ Check the name on the patient’s wristband
Module 3a: Chemotherapeutic Agents – Introduction

A. Review of the Cell Physiology and Infection


Bacteria
➢ Prokaryotes, single celled organism
➢ Classified according to shape and staining property
➢ Produce toxins that cause cell lysis
➢ Enzyme beta-lactamase that destroys beta lactam antibiotics

Cell nucleus
➢ genetic material
Cell membrane
➢ barrier
Lipoproteins
✓ lipids (phospholipid, glycolipids & cholesterol) and proteins
Receptor sites
✓ protein, reacts with specific chemicals outside the cell
Identifying markers
✓ surface antigen
Channels
✓ protein, pores

Cytoplasm
➢ within the cell membrane
Mitochondria
✓ energy production (power plant
Endoplasmic reticulum
✓ fine network, chemical reaction
Free ribosomes
✓ produce proteins
Golgi apparatus
✓ processing and packing (hormones, substances)
Lysosomes
✓ break down

Gram’s Stain
1. Gram-positive
➢ bacteria whose cell wall retains a stain, or resists decolorization without alcohol
2. Gram-negative
➢ bacteria whose cell walls lose a stain or are decolorized by alcohol

Aerobic or Anaerobic
1. Aerobic
➢ depend on oxygen for survival
2. Anearobic
➢ do not use oxygen to survive
Sources of Infection
a. Bacteria
b. Fungi
c. Virus
d. Protozoa
e. Rickettsia
f. Helminths

Chain of Infection

B. Introduction to Antimicrobial Agents


Anti-infective; Antibiotic; Antimicrobial
➢ drug that treats an infection caused by an organism (microbes, macrobiotic organisms)

✓ Antibacterial
✓ Antiviral
✓ Antifungal
✓ Antihelminthics
✓ Antiprotozal

ANTIBIOTIC
➢ Drug that targets any organism in the body including symbiotic microbes as well as micro-
and macro- organismal pathogens

General Principles of Therapy


1. To be useful antibiotic, a compound should inhibit the growth of bacteria without harming the
human host.
2. The drug should penetrate the body tissue in order to reach the bacteria

Bacteriostatic
➢ Directly eradicates / kills the invading microorganism

Bacteriostatic
➢ Substances that inhibit the growth of bacteria; weakens the pathogen sufficiently to allow the
patient’s immune system to complete the recovery process
Spectrum
➢ refers to the number/type of microorganism the antimicrobials is effective against.

Broad Spectrum
➢ Effective against many strains of microorganisms
➢ Frequently used to treat infections when the microorganism has not been identified by C&S
➢ Examples: tetracycline and cephalosporins

Narrow Spectrum
➢ Effective against few strains of microorganisms
➢ Since it is selective, it is more active against those single organisms than the broad-spectrum
antibiotics.
➢ Examples: penicillins and erythromycin

Extended Spectrum
➢ Modified to be effective against gram-positive organisms and also against a significant
number of gram-negative bacteria.

Antibacterial antimicrobial antibiotic – chemical produced by one kind of microorganism that


inhibits the groth or kill another so that would be their definition?

Use of Antibiotic Combinations


➢ Not be routinely prescribed or administered except for specific uncontrollable infections
➢ When there is a severe infection that persists unknown origin unsuccessfully treated with
several single antibiotics combination antibiotic
➢ Culture and sensitivity testing done first before beginning antibiotic therapy
➢ Combination of antibiotics may result to: additive, potentiation, or antagonistic

Resistance to Antibacterials
➢ Bacteria sensitive to drug
✓ Pathogen is inhibited or destroyed

➢ Bacteria resistant to drug


✓ Pathogen continues to grow, despite administration of drug;
✓ Natural or inherent resistance == occurs without previous exposure to the
antibacterial drug
✓ Acquired resistance == previous exposure to the antibacterial drug

➢ Cross-resistance occurs between antibacterial drugs that have similar actions, e.g. penicillins
and cephalosporins

2 types
Natural or inherent
Acquired resistance
Cross resistance

Drug resistance may occur by:


1. Mutation
➢ Bacteria undergo spontaneous mutation at a frequency of about 1 in 1016 cells.
Mutation may make the bacteria resistant to an antibiotic or it may not.

2. Adaptation
➢ Adaptation can take several routes. The bacteria may alter the uptake of the drug
by changes in their lipopolysaccharide coat. It may also improve transport system
that remove the drug from the cell. The bacteria may increase metabolism through
a pathway that bypasses the effect of antibiotic

3. Gene Transfer
➢ It occurs through plasmids and transposons.
➢ Plasmids are extrachromosomal genetic elements (pieces of RNA and DNA that
are not part of the choromosomes). These may code for enzymes that inactivate
antimicrobial. The plasmids are transferred from bacteria to bacteria by
conjugation and transduction
➢ Transposons are segments of genetic material with insertion sequences. They are
incorporated into the genetic makeup of bacteria and can also code for enzymes
that inactivate the antimicrobial.

Penicillinase
➢ Enzyme produced by the microorganism penicillin resistance.
➢ Penicillinase metabolizes penicillin G, causing the drug to be ineffective.

➢ Methicillin (Staphcillin) first penicillinase resistant penicillin developed in 1959 in response to a


resistance of S. aureus. In 1968, strains of S. aureus were beginning to become resistant to methicillin.
(miracle drug)

➢ Highly resistant bacteria methicillin-resistant Staphylococcus aureus (MRSA), became resistant not
only to methicillin, but to all penicillins and cephalosporins as well.
➢ The treatment of choice for MRSA is VANCOMYCIN

➢ Vancomycin-resistant Enterococcus faecium (VREF) cause death to many patients with weakened
immune systems.
➢ A strain of MRSA has been reported to be resistant to vancomycin (vancomycin-resistant
Staphylococcus aureus, or VRSA).
➢ LINEZOLID - effective against MRSA, VREF, penicillin-resistant Streptococci.

Antibiotic misuse
➢ antibiotics are taken unnecessarily (for viral infections, when no infection is present) or
incorrectly (skipping doses, not taking the full antibiotic regimen), improper dosage, omission
of surgical drainage

Role of the nurse


➢ Education on the proper use of antibiotics

Prophylactic Use of Antimicrobial


1. Pretreatment may prevent streptococcal infections in patients with a history of heart
disease. Patients may require years of treatment
2. Pretreating of patients undergoing dental extractions who have implanted prosthetic
devise, such as artificial heart valves, prevents seeding of the prosthesis
3. Pretreatment may prevent tuberculosis or meningitis among individuals who are close
contact with infected patients
4. Treatment prior to most surgical procedures can decrease the incidence of infection
afterwards. Effective prophylaxis is directed against the most likely organisms, not
eradication of every potential pathogen
C. Mechanisms of Action
Five Mechanisms of Actions of Antibacterial Agents
1. Inhibition of bacterial cell-wall synthesis, (penicillin)
2. Alteration of membrane permeability, (daptomycin)
3. Inhibition of protein synthesis, (aminoglycoside)
4. Inhibition of the synthesis of bacterial ribonucleic acid (RNA) and deoxyribonucleic acid
(DNA), (rifamycin)
5. Interference with metabolism within the cell (sulfonamide)

Pharmacokinetics
➢ Antibacterial drugs must not only penetrate the bacterial cell wall in sufficient concentration,
but also must have an affinity (attraction) to the binding sites on the bacterial cell.
➢ The time the drug remains at the binding sites increases the effect of the antibacterial action.
➢ This time factor is controlled by the pharmacokinetics (distribution, half-life, and elimination)
of the drug.
➢ Antibacterials that have a longer half-life usually maintain a greater concentration at the
binding site; therefore, frequent dosing is not required.
➢ Most antibacterials are not highly protein-bound, with a few exceptions (e.g., oxacillin,
ceftriaxone, cefoperazone, cefprozil, cloxacillin, nafcillin, clindamycin).
➢ Protein binding does not have a major influence on the effectiveness of most antibacterial
drugs.
➢ The steady state of the antibacterial drug occurs after the fourth to fifth half-lives, and the
drug is eliminated from the body, mainly through urine, after the seventh half-life.

Pharmacodynamics
➢ The drug concentration at the site or the exposure time for the drug plays an important role
in bacterial eradication.
➢ Antibacterial drugs are used to achieve the minimum effective concentration (MEC) necessary
to halt the growth of a microorganism thus the drug concentration should be remaining above
the MEC during the dosing interval for it to be effective.
➢ Duration of time for use of the antibacterial varies according to the type of pathogen, site of
infection, and immunocompetence of the host.
➢ Once-daily antibacterial dosing (e.g., aminoglycosides, macrolides, fluoroquinolones) has been
effective in eradicating pathogens and has not caused severe adverse reactions (ototoxicity,
nephrotoxicity) in most cases.

➢ The effectiveness of antibiotics also depends on the host’s defense mechanisms.


➢ Factors that influence the body’s ability to fight infection: age, nutrition, immunoglobulins,
white blood cells, organ function, and circulation
Common Side Effects among Antimicrobial
1. Allergy
2. Direct tissue damage
3. Superinfection
4. Nausea and Vomiting
5. Diarrhea

Common Adverse Reactions to Antimicrobial


1. Allergy or hypersensitivity
✓ Allergic reactions to drugs may be mild or severe.
✓ Examples of mild reactions are rash, pruritus, and hives.
✓ Example of a severe response is anaphylactic shock.
✓ Anaphylaxis results in vascular collapse, laryngeal edema, bronchospasm, and
cardiac arrest.
✓ Shortness of breath is frequently the first symptom of anaphylaxis.
✓ Severe allergic reaction generally occurs within 20 minutes.
✓ Mild allergic reaction is treated with an antihistamine; anaphylaxis requires
treatment with epinephrine, bronchodilators, and antihistamines.

2. Superinfection
➢ Superinfection is a new infection that appears during the course of treatment for
a primary infection
➢ Superinfections can occur in the mouth, respiratory tract, intestine, genitourinary
tract, or skin.
➢ Fungal infections frequently result in superinfections, although bacterial
organisms (e.g., Proteus,
➢ Pseudomonas, staphylococci) may be the offending microorganisms.
➢ Superinfections rarely develop when the drug is administered for less than 1 week.
➢ They occur more commonly with the use of broad-spectrum antibiotics.
➢ For fungal infection of the mouth, nystatin is frequently used.

3. Organ toxicity
➢ The liver and kidneys are involved in drug metabolism and excretion.
➢ Antibacterial may result in damage to these organs.
➢ For example, aminoglycosides can be nephrotoxic (as well as ototoxic).

Selection of the antimicrobial agents based on:


1. Sensitivity of the pathogen
2. Possibility of toxicity to the patient
3. Possibility of superinfection
4. Bactericidal or Bacteriostatic
5. Age of the client
6. Pregnancy and Lactation
7. Status of the client organ
8. Location of the infection
Nursing Considerations in the Use of Antimicrobial Agents:
1. Be aware of the potential dangers.
2. Be aware of the allergic reactions
3. Be aware of superinfection development
4. Identify the possible drug interaction if clients are taking other drugs
5. Explain the importance of full course treatment even through the patients feels well.
6. Educate the client about effects of medications and side effects.

- Inhibit cell wall synthesis


- Inhibit protein synthesis
- Piniclin
- Cephalosporins

AOL 3A:
TRUE or FALSE
1. Patients should take antibiotics as prescribed so they do not become resistant to antibiotics

2. Broad spectrum antibiotics kill more microbes than narrow spectrum antibiotics which increase the
nutrient available for resistant microbes, thus promoting resistant microbe reproduction.

3. Narrow spectrum antibiotics can promote overgrowth of normal flora that possess mechanisms for
resistance.

4. Infants do not have a developed blood brain barrier and drugs can enter the CNS more readily than in a
child or adult.

5. Frequent dosing is required for antibiotics that have longer half-life so as to maintain the drug serum
concentration above the minimum effective concentration.

6. Which part of the bacterial cell is not present in human cell?


➢ Cell Wall

7. Vancomycin is the treatment of choice for MRSA. What is its mechanism of action?
➢ Inhibition of bacterial cell-wall synthesis

8. Ruby is diagnosed to have rheumatic heart disease. She was given penicillin which she needs to take for a
long period of time. This is an example of what use of antibiotic?
➢ Nutritive B. Supplemental C. Prophylaxis D. Replacement

9. Selection of antimicrobial agent is based on all of the following, EXCEPT:


➢ Age of the patient B. Financial capability of the family C. Possibility of toxicity D. Location of the
infection

10. What is the primary role of the nurse in antibiotic therapy to prevent drug misuse?
➢ Health education on the proper use
➢ B. Promote the use of any antibiotic whenever the patient is not feeling well
➢ C. Encourage to use antibacterial for all types of infection
➢ D. Instruct the family that it is better to use higher doses of antibiotic than lower doses.
QUIZ 3A:
1. Where germs live
➢ Reservoir

2. How germs get in


➢ Portal of entry

3. Germs get around


➢ Mode of transmission

4. How germs get out


➢ Portal of exit

5. Next sick person


➢ Susceptible host

6. Inhibition of bacterial cell-wall synthesis


➢ Penicillin

7. Alteration of membrane permeability


➢ Daptomycin

8. Inhibition of protein synthesis


➢ Aminoglycoside

9. Inhibition of the synthesis of bacterial ribonucleic acid (RNA) and deoxyribonucleic acid (DNA
➢ Rifamycin

10. Interference with metabolism within the cell


➢ Sulfonamide

11. Which among the following is not present in human cell?


➢ Cell wall

12. Human cell has ribosomes of


➢ 80s, with subunits of 60s and 40s

13. Determine the drug of choice for MRSA


➢ Vancomycin

14. Which of the following drug must be given for VREF?


➢ Linezolid

15. Patient may discontinue taking antibiotics to prevent resistance


➢ False
Module 3b: Chemotherapeutic Agents – Antibacterial Agents

A. Inhibitors of Cell Wall Synthesis


General Features
➢ The final step in the synthesis of the bacterial cell wall is a cross-linking of adjacent peptidoglycan
strands by a process called transpeptidation.
➢ The penicillins and cephalosporins are structurally similar to the terminal portion of the
peptidoglycan strands and can compete for and bind to the enzymes that catalyze
transpeptidation and cross-linking. These enzymes are called penicillin-binding proteins.
➢ Interfere with these enzymes results in the formation of structurally weakened cell wall, oddly
shaped bacterial and ultimately DEATH.

β LACTAM
➢ All drugs in this group contains β-lactam ring in their structure
➢ The β-lactam ring makes it an effective antibiotic
➢ Some bacteria inactivate the β-lactam antibiotics by an enzyme (β lactamase) that opens the β-
lactam ring
➢ This leads to the inactivation of the antibiotic.
➢ The most common mode of drug resistance is plasmid transfer of the genetic code for the β
lactamase enzyme
➢ β lactamase specific for penicillin - penicillinase
➢ β lactamase specific for cephalosporin – cephalosporinase
➢ The inactivation of these two drugs by the β lactamase can be dealt with two approaches:
1. Give β lactamase inhibitors at the same time
2. Make chemical modification in the structure of the drug to make it more resistant to
inactivation

➢ Clavulanic Acid and Sulbactam are β lactamase inhibitors that are given together with the β-
lactam drugs to increase their effectiveness
✓ Ampicillin / Sulbactam = Unasyn®
✓ Amoxicillin / Clavulanate = Augmentin®
✓ Ticarcillin / Clavulanate
✓ Piperacillin / Tazobactam
I. PENICILLINS
➢ First isolated by Alexander Fleming from samples of mold (Penicillium notatum) that
contaminated a culture dish of Staphylococcus, producing a clear ring (lack of bacterial
growth) around the mold.
➢ Subgroups:
A. Natural Penicillin - penicillin G and penicillin V
B. Antistaphylococcal Penicillin - nafcillin, oxacillin, methicillin, dicloxacillin
C. Aminopenicillin - ampicillin, amoxicillin
D. Extended spectrum (antipsuedomonal) penicillin - piperacillin, ticarcillin

Pharmacokinetics
➢ Penicillin G is poorly absorbed when given orally, IM and IV injection is more
effective
➢ Amoxicillin well absorbed in GIT and excreted in urine
➢ Cloxacillin is partially absorbed in the GIT and excreted in bile

Indications
✓ Pneumonia
✓ Respiratory disease
✓ Urinary tract infection
✓ Syphilis
✓ Gonorrhea
✓ Meningitis
✓ Skin infections
✓ Some bone and joint infections
✓ Catheter infections
✓ Gram negative organisms affecting GI (shigella and salmonella)

Contraindication
✓ Allergic reaction
✓ Nursing mothers
✓ Neonates
✓ Renal, GI, hepatic dysfunction
✓ Bleeding disorders
✓ Myasthenia gravis
✓ Epilepsy

Side effects
✓ GI – N/V, flatulence, dark discoloration of the tongue, oral thrush, hepatitis
✓ Allergic reaction - most common cause of drug allergy
✓ Superinfection
✓ Injection sites reactions – inflammation, abscess, pain and irritation
✓ Hematologic reactions – bone marrow depression, hemolytic anemia
✓ CNS – lethargy, anxiety, depression, agitation, hallucination
Drug Interactions
➢ Probenecid – (down) renal elimination of penicillin
➢ Oral contraceptive – (down) effectiveness
➢ Anticoagulant – (up) risk of bleeding
➢ Other antibiotics – (down) effectiveness e.g. aminoglycosides
➢ Antacids – may impair the absorption of oral penicillins
➢ NSAIDs – risk of bleeding

1. Take amoxicillin and Amoxicillin clavulanate with meals to decrease GI distress


2. Take all penicillins with a full glass of water, 1 hour before or two hours
after meals to increaseabsorption
3. Oral penicillin G should be taken with water because acidic fruit juice can
inactivate the drug’santibacterial activity.
4. Do not discontinue the drug regimen before the complete prescription has been taken.
5. Avoid use of penicillins while breastfeeding.
6. Report significant side effects immediately like severe watery diarrhea and bloody
diarrhea
7. With older adults assessment of renal function is important

The most important adverse reaction of penicillin is HYPERSENSITIVITY REACTION


ALLERGIC REACTION
1. Immediate Reaction
➢ within 20 minutes after parenteral administration and consists of apprehension,
itching, paresthesis, wheezing, choking, fever, edema, and generalized urticarial
(hives). It can lead to hypotension, shock, loss of consciousness and death.
Mediated by IgE antibodies

2. Accelerated reaction
➢ appears 1 to 72 hours after drug administration and it consist mainly of urticarial

3. Late reaction
➢ more common with semi synthetic and appears 72 hours to several weeks after
drug administration. Consists mainly of skin rashes.

II. CEPHALOSPORINS
➢ Discovered by Giuseppe Brotzu from the fermentation product of fungus
Cephalosporium acremonium
➢ Bactericidal
➢ Because of their similarity, many persons allergic to penicillin are also allergic to
cephalosporins
➢ Five generations of cephalosporins
A. First Generation
➢ Cefazolin, Cephalexin, Cefadoxil, Cephradine
➢ Destroyed by β lactamase
➢ Aerobic gram-positive cocci (staphylococci & streptococci), some strains of E. coli,
K. pneumoniae, and P. mirabilis
➢ Moderate or poor activity against anaerobes, intracellular bacteria, or spirochetes
➢ Indication: Respiratory tract infection, skin and bone infections, genitourinary tract
infections

B. Second Generation
➢ Cefotetan, Cefoxitin, Cefuroxime, Cefaclor, Cefmetazole, Cefprozil, Cefonizid,
Cefamandole
➢ Some are resistant to β lactamase and possess a broader spectrum
➢ Potent against aerobic & facultative gram-negative bacteria, more potent against
E. coli, K. Pneumoniae & P. mirabilis than 1st generation cephalosporins, also
effective against Neisseria spp.
➢ Indication: Respiratory tract infection, septicemia, meningitis, gonorrhea, bone
and joint infection

C. Third Generation
➢ Cefotaxime, Ceftazidime, Ceftriaxone, Cefpodixime, Cefoperoxime, Cefdinir,
Cefixime
➢ Remain sensitive to AmpC β-lactamase
➢ Better penetration into body tissues, e.g. useful in treating pyelonephritis (kidney
infection) thus can be used as prophylaxis of infections in hospitalized patients
➢ Crosses the blood brain barrier (useful for treating meningitis)
➢ Indication: infections caused by enterobacter, P. aeruginosa and anaerobic
organisms

D. Fourth Generation
➢ Cefepime
➢ Adds activity against Pseudomonas
➢ Designed to target organisms with multiple drug resistance
➢ Indication: Infections caused by E. coli, Klebsiella, Proteus, Streptococci,
Staphylococci and P. aeruginosa

E. Fifth Generation
➢ Ceftaroline fosamil
➢ Newly FDA approved prodrug formulation of ceftaroline.
➢ Effective against gram-positive bacteria, and retains the activity of later-
generation cephalosporins with broad-spectrum activity against gram-negative
bacteria
➢ Indicated for treating MRSA, skin & soft tissue infections & community-acquired
pneumonia.
Pharmacokinetics
➢ Only few cephalosporins are administered orally, most of them are administered IM /
IV e.g. cephalexin, cefaclor, cefuroxime, cefdinir
➢ Cefazollin is administered IM / IV

Indications
➢ Ear and skin infection
➢ Respiratory tract and Urinary tract infection
➢ Septicemia
➢ Meningitis
➢ Osteomyelitis
➢ Prophylaxis for GI, GUT, bone and skin surgery

Side Effects
➢ GI - nausea, vomiting, diarrhea
➢ Increase in glucose values
➢ Anapylaxis may occur; alcohol may cause vomiting
➢ Nephrotoxicity
➢ Thrombocytopenia

Drug Interactions
➢ (down) Excretion with uricosuric drugs
➢ Cephalosporins taken with alcohol causes flushing, dizziness, headache, nausea,
vomiting and muscle cramps
➢ Effects of cefazolin and cefaclor when given with erythromycin or tetracycline
➢ Some cephalosporins (cefotetan and ceftriaxone) have anti-Vitamin K effects
(bleeding)
➢ Some cephalosporins cause acetaldehyde to accumulate

Eat cultured dairy products to discourage superinfections

III. CARBAPENEMS
➢ Synthetic analogs of (thienamycin),
➢ Naturally occurring product of Streptomyces cattleya that was discovered by Merck &
Co (Kahan et al, 1979).
➢ Broadest spectrum of β-lactams, effective against most gram-negative bacteria,
narrower spectrum against gram-positives, good activity against B. fragilis (an
anaerobe).
➢ Primarily reserved for treating bacteria with multi-drug resistance (MDR)

Imipenem
➢ Always combined with cilastin because it is otherwise rapidly destroyed by renal
dehydrogenase I
➢ Cilastatin is a DHP-I inhibitor thus t1/2 of imipenem is increase and its tissue penetration
Meropenem
➢ More stable to the renal peptidase and does not need the co-administration of
cilastatin
➢ Seizure risk at high plasma concentrations

IV. MONOBACTAMS
Aztreonam
➢ Only commercially available monobactam; excellent drug for aerobic gram-negative
bacteria including pseudomonas, but ineffective against gram positive organisms
➢ Synthetic drug based upon a simpler monobactam isolated from Chromobacterium
violaceum.
➢ Its spectrum of activity closely resembles that of aminoglycosides.
➢ Not absorbed from the GI tract
➢ May have cross allergic reaction with cephalosporins

V. POLYPEPTIDES
Vancomycin
➢ Effective against gram + organisms
➢ MOA: it disrupts the cell wall by binding to molecules that serve as precursors for cell
wall biosynthesis.
➢ Very poorly absorbed orally
➢ It is the “silver bullet” for methicillin-resistant staphylococci
➢ Major toxicity: renal failure
➢ It can cause a dose related ototoxicity that produces tinnitus, high tone deafness,
hearing loss and possible deafness, thrombocytopenia
➢ Rapid infusion of vancomycin can cause a constellation of disturbing effects—flushing,
rash, pruritus, urticaria, tachycardia, and hypotension— Red Man Syndrome – related
to release of histamine infuse vancomycin slowly (over 60 minutes or more).

Bacitracin
➢ Mixture of polypeptides that inhibit cell wall synthesis; used topically and causes
serious nephrotoxicity

B. Protein Synthesis Inhibitors (30s inhibitors and 50s inhibitors)


➢ The protein synthesis inhibitors bind to either the 30s or 50s ribosomal subunit and interfere with
the transcription of mRNA into protein

General Features
➢ Bacterial ribosomal subunits are different from mammalian ones
➢ Prokaryotes have 70s ribosomes each subunit consisting of small subunit is of 30s and the large
subunit is of 50s. Eukaryotes have 80s ribosomes each consisting of small subunit of 40s and
large subunit of 60s
➢ Function: They assemble amino acids to form specific proteins which is essential to carry out
cellular activity
➢ Resistance to these drugs is related to decreased uptake of the drugs or to altered ribosomal
subunits.
I. 30s Inhibitors
A. Aminoglycosides
➢ Broad spectrum antimicrobials however anaerobic bacteria are generally resistant
to them
➢ Used to treat serious infections against gram negative aerobic bacilli, as well as
gram positive bacteria
➢ Used to treat severe infections: sepsis, endocarditis, bacteremia
➢ The margin of safety with this drug is small - toxic concentration is only slightly
higher than the therapeutic concentration

These drugs all end in “-mycin” or “-micin” EXCEPT: amikacin


Gentamycin, Tobramycin, Amikacin, Kanamycin, Neomycin, Netilmicin, Streptomycin

Therapeutic Actions and Indications


➢ Bactericidal
➢ Inhibit protein synthesis in susceptible strains of gram-negative bacteria
➢ Irreversibly binds to 30s ribosome, causing code misreading; misread peptides
incorporate in cell membrane causing fissures
➢ Gram negative – Pseudomonas aeruginosa, E. coli, Proteus species, Klebsiella-
Enterobacter – Serratis group, Citrobacter species
➢ Gram positive = staphylococcus species
➢ Serious infections susceptible to penicillin when penicillin is contraindicated

Pharmacokinetics
➢ Poorly absorbed in the GI
➢ Rapidly absorbed after IM injection – peak within 1 hour
➢ Crosses the placenta and enters the breast milk
➢ Average t1/2 of 2 to 3 hours

Contraindication and Caution


➢ No absolute contraindication exists
➢ Known allergy
➢ Renal or hepatic disease (pre-existing)
➢ Pre-existing hearing loss
➢ Active infection with herpes or mycobacterial infections
➢ Myasthenia gravis
➢ Lactation
➢ Pregnancy category: D
Side Effects and Adverse Reaction
➢ NEPHROTOXICITY - r/t rapid uptake of the drug to proximal tubule (cells are
killed) -- reversible limited for less than 5 days of administration; more severe
in patients with acidosis and sodium deficiency
➢ OTOTOXICITY - Auditory and vestibular damage – tinnitus, deafness, vertigo,
unsteadiness of gait, high frequency hearing loss
➢ GI effects – nausea, vomiting, diarrhea, weight loss, stomatitis, hepatotoxicity
➢ Cardiac effects – palpitations, hypotension and hypertension
➢ Hypersensitivity – purpura, rash, urticarial, exfoliative dermatitis
➢ NEUROMUSCULAR TOXICITY - weakness and respiratory depression

Drug Interaction
➢ Drug resistance to aminoglycosides is common
➢ Loop diuretics – increase change of ototoxicity, nephrotoxicity, neurotoxicity
➢ Synergistic effect with penicillin, cephalosporin, carbenicillin, ticarcillin
➢ Increase effect of neuromuscular blocking agents

Amikacin Gentamycin Kanamycin


▪ Short- term IM or IV use for ▪ Ophthalmic, topical, IV, ▪ Parenteral and oral form
Gram (-) infections intrathecal, impregnated beads ▪ Hepatic coma
▪ Nephrotoxicity and ototoxicity on surgical wire, liposomal ▪ Adjunctive therapy to decrease
▪ Use only as long as necessary injection bacterial flora
▪ Wide variety of infections, ▪ Not to be used longer than 7 to
pseudomonal diseases and once- 10 days
rare infections in AIDS
▪ Prototype drug of
aminoglycosides
Neomycin Streptomycin Tobramycin
▪ Milder aminoglycoside ▪ Very toxic to the cranial nerve ▪ Short-term IM or IV treatment of
▪ Suppress GI bacteria VIII and kidney very serious infections
preoperatively and hepatic coma ▪ 4th drug for TB ▪ Ophthalmic form
▪ Topical: skin wounds and ▪ Severe infections sensitive to ▪ Can cause c. difficile associated
infection streptomycin and no less toxic diarrhea
drug can be used

Nursing Considerations:
1. Assess: allergy, history of renal or hepatic disease, pre-existing hearing loss, active
infection, neuromuscular disease, pregnancy
2. Implementation: C&S; patient compliance with therapy; monitor signs and
symptoms of infection, signs of nephrotoxicity and bone marrow suppression;
safety measures for CNS affectation; small frequent feedings; mouth care, ice
chips; hydration

B. Tetracyclines
➢ Developed as semisynthetic antibiotics based on the structure of a common soil
mold
➢ Composed of four rings (origin of name)
➢ Broad spectrum antibiotics
➢ They accumulate in the cytoplasm by energy dependent transport system (not
present in mammalian cells)
➢ Newer tetracyclines to increase absorption and tissue penetration
➢ Limited use because of widespread resistance
➢ When penicillin is contraindicated, this is often used.
End in “-cycline”
Tetracycline, Chlortetracycline, Demeclocycline, Doxycycline, Minocycline,
Oxytetracycline

Demeclocycline
➢ Not administered with food
➢ Stimulates urine flow - excessive urination, thirst, and tiredness.
➢ Employed therapeutically to promote urine production in patients suffering
from the syndrome of inappropriate (excessive) secretion of antidiuretic
hormone.

Doxycycline
➢ Given on an empty stomach
➢ First-line drug for Lyme disease, anthrax, chlamydial infections (urethritis,
cervicitis, and lymphogranuloma venereum), and sexually acquired proctitis (in
combination with ceftriaxone).

Minocycline
➢ Damage the vestibular system, causing unsteadiness, lightheadedness, and
dizziness.
➢ Reduce symptoms of arthritis

Therapeutic Actions and Indications


➢ Bacteriostatic
➢ MOA: inhibit protein synthesis leading to inability of the bacteria to multiply
➢ Affected protein in microorganisms is similar to human – maybe toxic in high
doses
➢ First line drug for: (1) rickettsial diseases (e.g., Rocky Mountain spotted fever,
typhus fever, Q fever);
➢ (2) infections caused by Chlamydia trachomatis (trachoma, lymphogranuloma
venereum, urethritis, cervicitis); (3) brucellosis; (4) cholera; (5) pneumonia
caused by Mycoplasma pneumoniae; (6) Lyme disease; (7) anthrax; and (8)
gastric infection with H. pylori.
➢ Use topically for the treatment of acne vulgaris

Pharmacokinetics
➢ Absorbed adequately but not completely in the GI
➢ Absorption is affected by food, iron, calcium and other drugs in the stomach
➢ Poorly penetrated the CSF, readily cross placenta and enter the fetal
circulation, breast milk
➢ Concentrated in the liver
➢ Excreted in the urine
➢ Not used in pregnant and lactating women and children younger than 8 years
old

Contraindication and Caution


➢ Known allergy to tetracycline
➢ Pregnancy and lactation
➢ Use in caution in children younger than 8 years old
➢ Hepatic and renal dysfunction
Adverse Effects
➢ GI: nausea, vomiting, diarrhea
➢ Superinfection such as vaginitis and stomatitis (diarrhea may indicate
potentially life-threatening superinfection)
➢ Photosensitivity
➢ Discolored teeth
➢ Retarded bone growth
➢ Hepatic and renal toxicity
➢ Thrombophlebitis with IV administration

Drug – Drug; Drug – Food Interaction


➢ Food impairs absorption of tetracycline except: doxycycline and minocycline
➢ Penicillin + tetracycline = penicillin becomes less effective
➢ Digoxin toxicity increases when given with tetracycline
➢ Oral contraceptives become less effective
➢ Not administered together with (1) calcium supplements, (2) milk products
(because they contain calcium), (3) iron supplements, (4) magnesium-
containing laxatives, and (5) most antacids (because they contain magnesium,
aluminum, or both)

Nursing Considerations:
Instruct patient to
a. avoid sun exposure (abnormal sunburn reaction, photosensitivity)
b. avoid medication if pregnant or breastfeeding, and under 8 years old (staining of
teeth, retardation of bone growth)
c. administer in empty stomach (1 hour before meals or 2-3hours after meals) d.) not
to take on bedtime (risk for erosive esophagitis if taken at bedtime)

II. 0s Inhibitor
A. Macrolides
➢ Erythromycin and its relatives are of particular use in the treatment of patients
with mycoplasma infections, pneumonia, Legionnaire’s disease, chlamydial
infections, diphtheria, and pertussis.
➢ Erythromycin - drug of choice in Legionella, Campylobacter, Mycoplasma,
Bordetella

End in “-romycin” and mostly in “-thromycin”


Erythromycin, Azithromycin, Clarithromycin

Therapeutic Actions and Indications


➢ Bacteriostatic, can be bactericidal in large doses
➢ MOA: bind to bacterial cell membrane and change protein function; binds to
50s ribosome

Pharmacokinetics
➢ Absorbed in variable rates in the GI
➢ Excreted in the feces, in the urine (unchanged drug)
➢ Crosses the placenta and enters the breast milk

Contraindication and Caution


➢ Known allergy to macrolides (cross sensitivity)
➢ Pregnancy and lactation: B
➢ Caution: Hepatic dysfunction, renal disease
Side Effect / Adverse Effect
➢ Few serious sides effect
➢ GI upset is a common side effect

Drug – Drug / Drug – Food Interaction


➢ Digoxin – increased serum level of digoxin
➢ Oral anti-coagulants, theophyllines, carbamazepine, corticosteroids –
increased effects of these drugs
➢ Decreased absorption of oral macrolides

Nursing Considerations:
Instruct patient to
a. Take medication with full glass of water one hour before or 2 to 3 hours after meals

B. Chlorampenicol
➢ It is a broad-spectrum antibiotic, effective against aerobic and anaerobic bacteria
except Pseudomonas aeruginosa
➢ Pharmacokinetics: orally absorbed, penetrates the CSF, and is inactivated in the
liver by conjugation
➢ It is associated with bone marrow depression and aplastic anemia (FATAL)
➢ It can produce gray baby syndrome (FATAL) abdominal distention, vomiting,
cyanosis, hypothermia, decreased respiration, vasomotor collapse

C. Lincosamides
➢ Clindamycin and Lincomycin
➢ Similar to macrolides but are more toxic
➢ React at almost the same site in bacterial protein synthesis

Clindamycin
➢ Penetrates most tissues, including bone. It has activity against anaerobic
organisms
➢ If superinfection with C. difficile is diagnosed, clindamycin should be
discontinued and the patient given oral vancomycin or metronidazole, which
are the drugs of choice for eliminating C. difficile from the bowel.

Linezolid
➢ First-in-class oxazolidinone antibiotic.
➢ Activity against multidrug-resistant gram-positive pathogens, including
vancomycin-resistant enterococci (VRE) and methicillinresistant
Staphylococcus aureus (MRSA).
➢ Five approved indications:
a. Infections cause by VRE
b. Healthcare-associated pneumonia caused by Staph. aureus
(methicillin-susceptible and methicillin-resistant strains) or Strep.
pneumoniae (penicillin-susceptible strains only)
c. Community-associated pneumonia (CAP) caused by Strep.
pneumoniae (penicillin-susceptible strains only)
d. Complicated skin and skin structure infections caused by Staph.
aureus (methicillin-susceptible and methicillinresistant strains),
Strep. pyogenes, or Strep. Agalactiae
e. Uncomplicated skin and skin structure infections caused by Staph.
aureus (methicillin- susceptible strains only) or Strep. pyogenes
➢ Adverse effect: reversible myelosuppression, manifesting as anemia,
leukopenia, thrombocytopenia, or even pancytopenia

C. Interference of Metabolism within the cell (Folate Antagonists)


A. Sulfonamides
➢ Among the oldest anti-infectives; first drugs available for systemic treatment
of bacterial infection
➢ Also known as sulfa drugs
➢ Inexpensive and effective treatment for urinary tract and trachoma infections
➢ Most effective in combination with other drugs (trimethoprim +
sulfamethoxazole = cotrimoxazole) - useful for urinary tract infection and
pneumocystis carinii pneumonitis
➢ Broad spectrum of Gram positive and Gram-negative microorganisms
➢ Silver Sulfadiazine and Mafenide - suppress bacterial colonization in patients
with second- and third-degree burns. Mafenide acts by the same mechanism
as other sulfonamides. Silver sulfadiazine acts by the release of free silver.
Local application of mafenide is frequently painful, but application of silver
sulfadiazine is usually pain free.

Sulfamethoxazole, Trimethorprim, Cotrimoxazole, Sulfacetamide,


Sulfadiazine, Sulfapryridine, Sulfasalazine, Sulfisoxazole

Therapeutic Actions and Indications


➢ Bacteriostatic
➢ MOA: Competitively block para-aminobenzoic acid (PABA) to prevent the
synthesis of folic acid that synthesize their own folate for production of RNA
and DNA. Bacteria cannot absorb folic acid, but must make it from PABA,
pteridnine and glutamate
➢ Sulfonamides and Trimethoprim inhibit synthesis of folate at two different
sites.
➢ The sulfonamides are structurally similar to PABA and block the incorporation
of PABA into dihyropteroic acid.
➢ Trimethoprim prevents reduction of dihydrofolate to tetrahydrofolate by
inhibiting the enzyme dihydrofolate reductase.
➢ Indication: Urinary tract infection, sexually transmitted infection, nocardiosis,
trachoma, infections caused by c. trachomatis, h. influenza, e. coli, p. mirabilis
Pharmacokinetics
➢ Absorbed in the gastrointestinal tract
➢ Metabolized in the liver
➢ Excreted in the urine
➢ Teratogenic – not used during pregnancy and lactation

Contraindication and Caution


➢ Known allergy to sulfonamides, sulfonylurea, thiazide diuretics
➢ Pregnancy and Lactation
➢ Caution: Renal disease and kidney stones

Adverse Effects
➢ GI: nausea, vomiting, diarrhea, abdominal pain, anorexia, stomatitis, hepatic
injury
➢ GU: crystalluria, hematuria, proteinuria
➢ CNS: headache, dizziness, vertigo, ataxia, convulsion, depression
➢ Bone marrow depression
➢ Photosensitivity and rash
➢ Most severe hypersensitivity response - Stevens-Johnson syndrome
(widespread lesions of the skin and mucous membranes, combined with fever,
malaise, and toxemia)

Drug Interaction
➢ Potentiation of anti-coagulants and oral antidiabetics
➢ Antagonism: local anesthetics – procaine inhibits antibacterial effect; digitalis
and phenytoin

Nursing Considerations:
Instruct patient to
a. Drink large amount of fluid, discontinue at first sign of rash, avoid exposure to
sunlight,
b. Ingest with food delays absorption
Silver sulfadiazine can cause a blue-green or gray skin discoloration, avoid facial application

B. Trimethoprim
➢ Suppresses synthesis of tetrahydrofolic acid.
➢ Active against a broad spectrum of microbes
➢ Absorbed rapidly and completely from the gastrointestinal (GI) tract, lipid
soluble, widely distributed
➢ Most frequent adverse effects are itching and rash. GI reactions (e.g.,
epigastric distress, nausea, vomiting, glossitis, stomatitis) occur occasionally.

D. Alteration of Membrane Permeability


Peptides
➢ Derived from cultures of Bacillus subtilis
➢ Interfere with bacterial cell membrane funation

A. Bacitracin
➢ Polypeptide structure and acts by inhibiting bacterial cell wall synthesis and
damaging cell wall membrane
➢ Bacteriostatic or bactericidal
➢ Not absorbed via GIT if given orally and it is excreted in the feces.
➢ OTC bacitracin ointment available for application in the skin.
➢ Side effects: redness, rash, nausea, vomiting
➢ Severe adverse reactions: renal damage and oxtotoxicity

B. Daptomycin
➢ Naturally-occurring lipopeptide antibiotic.
➢ Found in the soil bacterium Streptomyces roseosporus.

Mechanism of Action
➢ Bind or insert into the outer membrane of gram-positive bacteria.
➢ The binding and integration of daptomycin into the cell membrane is calcium
dependent.
➢ Calcium ions cause a conformational change in daptomycin, augmenting its
amphipathicity (hydrophilic head group and hydrophobic tail group), leading to
incorporation into the cell membrane.
➢ This binding causes rapid depolarization, resulting in a loss of membrane potential
leading to inhibition of protein, DNA and RNA synthesis, which results in bacterial cell
death.

Indication
➢ Complicated skin and skin structure infections caused by susceptible strains of Gram-
positive microorganisms

C. Polymyxin B
➢ It is derived from the bacterium Bacillus polymyxn

Mechanism of Action
1. It alters the permeability of the bacterial outer membrane by binding to a
negatively charged site in the lipopolysaccharide layer, which has an
electrostatic attraction for the amino groups positively charged in the
cyclic peptide part.
2. The fatty acid portion dissolves in the hydrophobic region of the
cytoplasmic membrane and alters the integrity of the membrane.
3. Leakage of cellular molecules, inhibition of cellular respiration.
4. It binds and inactivates endotoxin.
5. Relative absence of selective toxicity: non-specific for cell membranes of
any type, highly toxic.

Indication
➢ Urinary tract infections and meningitis caused by Pseudomonas aeruginosa
and Haemophilus influenza

Side Effects / Adverse Effects


➢ High serum levels of polymyxin – nephrotoxicity (reversible when serum drug
level decreases) and neurotoxicity (reversible when drug is discontinued)
➢ Injections into muscle may result in significant pain.
➢ Fungal infections, anaphylaxis, and muscle weakness
E. Inhibition of the Synthesis of Bacterial RNA and DNA
A. Quinolones
➢ Antibiotics obtained by chemical synthesis which is derived from quinolone
➢ Nalidixic acid - first quinolone used
➢ Considered as broad spectrum

Mechanism of Action
✓ They inhibit DNA gyrase and thus, DNA synthesis. DNA gyrase is the bacterial
enzyme that is responsible for unwinding and supercoiling of the DNA
✓ This is the only class of antibacterial that inhibit DNA replication

B. Flouroquinolones
➢ Quinolone with fluorine atom
➢ A broad antibiotic class with both Gram positive and Gram-negative coverage
➢ Relatively new class of antibiotics
➢ Synthetically made

Therapeutic Actions and Indications


✓ Bactericidal
✓ MOA: Enter the bacterial cell wall by passive diffusion through channels in the cell
membrane
✓ Interfere with the action of the DNA enzymes necessary for the growth and
reproduction of bacteria
✓ Leads to cell death
✓ It is proven for the prevention of anthrax infection and effective for typhoid fever
✓ Primarily to treat patients with urinary tract infections
✓ Can be given for respiratory tract and skin infection

For both Quinolones and Flouroquinolones….


Generation Medication General Clinical Indication
First Nalidix Acid, Cinoxacin Uncomplicated urinary tract infections
Second Norfloxacin, Uncomplicated and complicated urinary tract infections and
Lomefloxacin, Enoxacin, pyelonephritis, sexually transmitted disease, prostatitis, skin and soft
Ofloxacin, Ciprofloxacin tissue infections
Third Levofloxacin, Acute exacerbations of chronic bronchitis, community – acquired
Sparfloxacin, pneumonia
Gatifloxacin, Moxifloxacin
Fourth Trovafloxacin Same as for first-, second-, and third-, generation agents (excluding
complicated urinary tract infections and pyelonephritis) plus intra-
abdominal infections, nosocomial pneumonia, pelvic infections

Pharmacokinetics
➢ Absorbed in the GI
➢ Metabolized in the liver
➢ Excreted in the urine and feces
➢ Crosses the placenta and enters the breast milk

Contraindication and Caution


➢ Older adults with GI disease or arteriosclerosis
➢ Children and adolescents – potential cartilage damage
➢ Strenuous exercise during and several weeks after the therapy – potential for tendon
rupture
➢ Pregnancy and lactation: C
➢ Severe renal impairment
➢ Seizure disorders
➢ Cardiac disease
➢ Myasthenia gravis - Ciprofloxacin and other fluoroquinolones can exacerbate muscle
weakness

Adverse Effects
➢ GI: nausea, vomiting, diarrhea, anorexia, abdominal pain, colitis
➢ CNS: headache, dizziness, lethargy or paresthesias
➢ Crystalluria
➢ Superinfection
➢ Phototoxicity
➢ Possible cartilage or tendon damage

Drug Interaction
Theophylline
✓ potentiate serious CNS effects, cardiac arrest, respiratory failure

Probenecid
✓ increased blood levels of Ciprofloxacin

Antacids
✓ decreased absorption

Coumadin
✓ increased risk of bleeding

Preparations containing Fe, Mg, Zn, Ca


✓ decreased absorption

Sucralfate
✓ contains Al ions, decreased absorption

Nursing Considerations:
Instruct patient to:
a. Not to take other medications without physician’s approval
b. Drink liberal quantities of fluids
c. Restrict caffeine intake – CNS effects
d. Avoid excessive exposure to sunlight
e. Avoid strenuous activities
f. Assess patients for GI and CNS effects

C. Antituberculosis Drugs
➢ Used to treat tuberculosis infection
➢ Commonly used in combination
➢ Drugs are curative but likewise can halt the progression of a mycobacterial infection

Pharmacokinetics
➢ Administered orally – well absorbed form the GI tract and widely distributed in the
body
➢ Metabolized by the liver
➢ Excreted mainly in the urine
Pharmacodynamics
➢ Specific to mycobacteria
➢ Tuberculostatic (isoniazid and ethambutol)
➢ Tuberculocidal – rifampicin
➢ Bacterial resistance – isoniazid and rifampicin

Mechanism of Action
First – Line Drugs
✓ Isoniazid (INH) - it inhibits synthesis of mycolic acid coating of the bacterium
✓ Rifampicin (RFM) - inhibits RNA synthesis by formation of a stable complex with the
DNA dependent RNA polymerase
✓ Ethambutol (EMB) - inhibits cellular metabolism
✓ Pyrazinamide (PZA) - both bactericidal and bacteriostatic

Second – Line Drugs


✓ Ethionamide - prevents cell division
✓ Rifapentin - alters DNA and RNA activity causing cell death
✓ Aminosalicylic acid
✓ Kanamycin
✓ Quinolones
✓ Rifabutin
✓ Viomycin

Third – Line Drugs


✓ Capreomycin – unknown MOA
✓ Cycloserine – inhibits cell wall synthesis and leads to cell death

Multi-drug Therapy
➢ Use of drug in combination
➢ Helps decrease the emergence of resistant strains and to affect bacteria at various
phase of their long and slow life cycle
➢ MDT is recommended because of the following reasons
1. Resistance in M. tuberculosis occurs because of spontaneous mutations.
2. Each mutational event confers resistance to only one drug.
3. Mutations conferring resistance to a single drug occur in about 1 of every
100 million (108) bacteria.
4. The bacterial burden in active TB is well above 108 organisms but far below
1016.
5. M. tuberculosis grows slowly, and hence treatment is prolonged.

Isoniazid Rifampicin
▪ Drug of choice for chemoprophylaxis in recent ▪ Effective also with gram (+) and (-) microorganism
convertors ▪ It is metabolized in the liver and is potent reducer of
▪ (-) TB test in the past then 1 year later the test is (+) the P-450 enzymes – hepatitis
➔ recent convertor ▪ Red orange secretions – urine, feces, saliva, sweat
▪ INH for 6 to 12 months as long as there is no evidence tears (even get into contact lenses)
of clinical disease, such as (+) CSR
▪ Adverse effects: peripheral neuritis, Vitamin B6
depletion, hepatotoxicity
Pyrazamide Ethambutol
▪ Effective only against M. tuberculosis ▪ Adverse effect: optic neuritis (loss of central vision and
▪ Increases level of serum uric acid impaired red-green discrimination)
▪ Hepatotoxic ▪ Increases level of serum uric acid leading to gout
Steptomycin
▪ Ototoxicity, nephrotoxicity
Nursing Considerations:
Instruct patient to:
a. Take rifampicin on empty stomach
b. Finish the recommended drug regimen course
c. Rifampicin causes discoloration of urine, feces, sputum, sweat and tears
d. Not to wear contact lens
e. Avoidance of alcohol

D. Leprostatic Agents
Dapsone
➢ Mainstay leprosy treatment for many years
➢ Inhibits folate synthesis
➢ P. carinii pneumonia in AIDS
➢ Absorbed rapidly and nearly completely from the GI tract, widely distributed
to tissues and body fluids, undergoes hepatic metabolism followed by
excretion in the urine. The average half-life is 28 hours.
➢ Most common effects are GI disturbances, headache, rash, and a syndrome
that resembles mononucleosis, hemolytic anemia occurs occasionally, severe
reactions are usually limited to patients with profound glucose-6-phosphate
dehydrogenase deficiency. Rare reactions include agranulocytosis, exfoliative
dermatitis, and hepatitis.

Clofazimine
✓ Derivative of phenazine
✓ Used in dapsone – resistant leprosy.
✓ It binds to DNA
✓ Bioavailability is increased with food
✓ Very long half-life of elimination about 10 days because it is stored in the
tissues.
✓ Adverse effects: reddish discoloration of skin, conjunctiva and tears

E. Urinary Antiseptics
➢ Nitrofurantoin - broad-spectrum antibacterial drug, producing bacteriostatic effects
at low concentrations and bactericidal effects at high concentrations
➢ Therapeutic levels are achieved only in urine.
➢ Nitrofurantoin can cause serious adverse effects.
➢ Injures bacteria by damaging DNA, to damage DNA, the drug must first undergo
enzymatic conversion to a reactive form.
➢ Indicated for acute infections of the lower urinary tract caused by susceptible
organisms.
➢ Can be used for prophylaxis of recurrent lower UTI.
➢ Not recommended for infections of the upper urinary tract.

F. Additional Antibacterial Agents


Metronidazole
➢ Used for protozoal infections and infections caused by obligate anaerobic
bacteria.
➢ MOA: To exert bactericidal effects, metronidazole must first be taken up by
cells and then converted into its active form; only anaerobes can perform the
conversion. The active form interacts with DNA to cause strand breakage and
loss of helical structure, effects that result in inhibition of nucleic acid synthesis
and, ultimately, cell death.
➢ Indication: Infections caused by anaerobic bacterial infections of the CNS,
abdominal organs, bones and joints, skin and soft tissues, and genitourinary
tract.

Daptomycin
➢ First representative of a new class of antibiotics, the cyclic lipopeptides.
➢ The drug has a unique mechanism and can rapidly kill virtually all clinically
relevant gram-positive bacteria, including methicillin-resistant Staph. aureus.
➢ The drug is given once daily by IV infusion, and there is no need to monitor its
plasma level
➢ Inserts itself into the bacterial cell membrane and thereby forms channels that
permit efflux of intracellular potassium (and possibly other cytoplasmic ions).
Loss of intracellular ions has two effects. First, it depolarizes the cell
membrane. Second, it inhibits synthesis of DNA, RNA, and proteins, and
thereby causes cell death.

Rifampin
➢ Broad-spectrum antibacterial agent employed primarily for tuberculosis
➢ It is also used against several non-tuberculous infections.
➢ Useful for treating asymptomatic carriers of Neisseria meningitidis, but is not
given to treat active meningococcal infection.

Rifaximin
➢ Inhibits bacterial DNA–dependent RNA polymerase and thereby inhibits RNA
synthesis, resulting in inhibition of protein synthesis and subsequent bacterial
death.
➢ Three approved uses: Traveler’s diarrhea caused by E. coli in patients at least
12 years old; Prevention of hepatic encephalopathy (brain injury) in patients
with chronic liver disease; Irritable bowel syndrome with diarrhea

Fidaxomicin
➢ Narrow-spectrum bactericidal, macrocyclic antibiotic indicated only for
diarrhea associated with CDI.

AOL 3B:

Identify the specific classification of the following drugs. If it is a cephalosporin, identify the generation
Answer Item Drug Answer Item Drug
1 Rifampicin 11 Cephalexin
2 Cefipime 12 Aztreonam
3 Gentamycin 13 Azithromycin
4 Amoxicillin 14 Ofloxacin
5 Clindamycin 15 Clofazimine
6 Sulfamethoxazole 16 Oxacillin
7 Cefaclor 17 Dapsone
8 Clarithromycin 18 Streptomycin
9 Doxycycline 19 Vancomycin
10 Meropenem 20 Cefixime
1. Amoxicillin is prescribed for a patient who has a respiratory infection. The nurse is teaching the patient
about this medication and realizes that more teaching is needed when the patient makes which
statement?
➢ This medication should not be taken with food.

2. A patient is taking a cephalosporin. The nurse anticipates which appropriate nursing intervention(s) for
this medication? Select all that apply.
➢ Monitoring renal function studies
➢ Monitoring liver function studies
➢ Infusing intravenous medication over 30 minutes
➢ Monitoring the patient for mouth ulcers

3. Penicillin G has been prescribed for a patient. Which nursing intervention(s) should the nurse perform for
this patient? Select all that apply.
➢ Collect culture and sensitivity before the first dose.
➢ Monitor the patient for mouth ulcers.
➢ Have epinephrine on hand for a potential severe allergic reaction.

4. A patient is prescribed daptomycin. Which action(s) should the nurse implement? Select all that apply.
➢ Monitor blood values for toxicity.
➢ Dilute in 50 to100 mL of normal saline and administer intravenously over 30 minutes.
➢ Monitor the patient for allergic reactions such as rhabdomyolysis.
➢ Culture the infected area before administering the first dose.

5. A patient is taking azithromycin. Which nursing intervention(s) would the nurse plan to implement for this
patient? Select all that apply.
➢ Monitor periodic liver function tests.
➢ Instruct the patient to report any hearing loss.
➢ Instruct the patient to report evidence of superinfection.
➢ Teach the patient to take oral drug 1 hour before or 2 hours after meals.

6. For which serious adverse effect should the nurse closely monitor a patient who is taking lincosamides?
➢ Clostridium difficile–associated diarrhea

7. The nurse enters a patient’s room to find that his heart rate is 120, his blood pressure is 70/50, and he has
red blotching of his face and neck. Vancomycin is running intravenous piggyback. The nurse believes that
this patient is experiencing a severe adverse effect called red man syndrome. What action will the nurse
take?
➢ Stop the infusion and call the health care provider.

8. A patient is receiving tetracycline. Which advice should the nurse include when teaching this patient about
tetracycline?
➢ Take sunscreen precautions when at the beach.

9. A patient is taking levofloxacin. What does the nurse know to be true regarding this drug?
➢ An adverse effect is dysrhythmia.

10. Which instruction(s) will the nurse include when teaching patients about gentamicin? Select all that apply.
➢ Patients should report any hearing loss.
➢ Patients should use sunscreen when taking gentamicin.
➢ Patients are monitored for mouth ulcers and vaginitis.
➢ Patients should increase fluid intake.
QUIZ 3B:

1. Your patient who is taking streptomycin has developed tinnitus. This can be a sign of:
➢ Ototoxicity

2. Cross allergic reaction can occur in which of the following drugs?


➢ Flouroquinolones and Monobactam

3. Which medication can cause staining of the teeth that is why it is not given in children below 8years old?
➢ Tetracycline

4. If penicillin will be given with probenecid, there will be __ renal elimination of penicillin
➢ Decrease

5. Gray baby syndrome is associated with what drug


➢ Chloramphenicol

6. What will the nurse routinely monitor in 65-year-old patient taking amikacin?
➢ Hearing problems

7. Cefixime belongs to which generation of cephalosporins?


➢ Third

8. All of the following are true about penicillin, except:


➢ Pen g is well absorbed orally

9. The nurse assesses a patient who is taking gentamicin. What assessment findings should be cause for serious
concern?
➢ Ototoxicity

10. Determine the most common cause of penicillin drug allergy


➢ Allergic reaction

11. When your client was diagnosed with lyme disease, which of the following drug is expected to be prescribed?
➢ Doxycycline

12. How penicillin and cephalosporin work?


➢ Inhibit cell wall synthesis

13. Why is imipenem is combined with Cilastin?


➢ To reduce side effects

14. When taking vancomycin, which of the following must be assessed?


➢ Liver function

15. A patient is scheduled to receive ceftriaxone (Rocephin) for klebsiella pneumoniae. What will the nurse teach
the patient about this medication?
➢ It is given IM or IV only

You might also like